Вы находитесь на странице: 1из 108

curso de post-grado para profesores especialidad en matematica GEOMETR EUCLIDEANA IA

Equipo de Dise o: n Nahomy Jhopselyn Hernndez Cruz a Gabriel Alexander Chicas Reyes Eduardo Arnoldo Aguilar Caas n Hctor Enmanuel Alberti Arroyo e Ernesto Amrico Hidalgo Castellanos e Juan Agustn Cuadra Claudia Patricia Corcio Lpez de Beltrn o a Carlos Mauricio Canjura Linares Oscar Armando Hernndez Morales a Aarn Ernesto Ramrez Flores o 15 de marzo de 2010

Indice
1. Angulos entre paralelas. 2. Tringulos: Teoremas Fundamentales. a 3. Congruencia de Tringulos. a 4. Cuadrilteros: Clasicacin y Propiedades. a o 5. Angulos en la Circunferencia. 6. Teorema de Thales y su rec proco. Semejanza de Tringulos. a 7. Puntos y Rectas Notables del Tringulo. a 8. Solucin a Problemas Selectos. o 2 7 16 24 30 46 58 72

1.

Angulos entre paralelas.

ANGULOS
Denimos como ngulo a la gura geomtrica formada por dos rayos (o semirrectas) distintas a e que tienen el mismo origen. Ese origen se llama vrtice del ngulo. Al ngulo de vrtice O y e a a e rayos OA y OB se le denota AOB. Dos ngulos AOB y BOC son adyacentes si y slo si tienen un lado comn OB y los lados a o u no comunes OA y OC estn en semiplanos distintos, determinados por el lado comn. a u Bisectriz de un angulo es la semirrecta que lo divide en dos angulos adyacentes iguales. Dos ngulos son: a Congruentes o Iguales: si tienen igual medida. Suplementarios: si su suma es 180. Complementarios: si su suma es 90. Por otra parte, dos rectas en el plano pueden ser secantes o paralelas,1 dependiendo si se cortan o no; adems, si las rectas son secantes, el punto de corte es unico, y denen cuatro ngulos, a a que se agrupan por parejas en ngulos opuestos por el vrtice (las parejas de angulos tales que a e uno est formado por la prolongacin de los lados del otro). a o Los angulos opuestos por el vrtice son iguales (Justique), por lo que dos rectas secantes for e man cuatro angulos que denen dos parejas de ngulos iguales, y si tomamos un miembro de a cada pareja, se tienen dos angulos suplementarios. En particular, si las rectas son secantes y forman cuatro angulos iguales, sern llamadas rectas perpendiculares,2 y los ngulos as gene a a rados son llamados ngulos rectos. Y como es muy conocido, un ngulo agudo es aquel cuya a a medida es menor a la de un angulo recto, y un ngulo obtuso es aquel cuya medida es mayor a que un angulo recto; en particular, un angulo obtuso ser llamado ngulo llano si su medida es a a el doble que la de un ngulo recto. a

ANGULOS ENTRE PARALELAS


Al intersecar un par de rectas paralelas por una recta llamada transversal o secante, se forman los siguientes tipos de ngulo: a Angulos Correspondientes: Son dos ngulos no adyacentes situados en el mismo lado de a la secante, uno en el interior y otro en el exterior de las paralelas. Angulos Alternos Internos: Son dos angulos no adyacentes situados en el interior de las paralelas, y en distintos lado de la secante.
1 2

Si la recta AB es paralela a la recta CD, se denota AB CD. Si la recta AB es perpendicular a la recta CD, se denota AB CD.

Angulos Alternos Externos: Son dos angulos no adyacentes situados en el exterior de las paralelas, y en distintos lado de la secante. Angulos Conjugados: Son los angulos no adyacentes situados uno en el interior y el otro en el exterior de las rectas paralelas y del mismo lado de la secante. Las propiedades fundamentales de los ngulos entre paralelas son: a 1. Los angulos correspondientes son iguales entre s . 2. Los angulos alternos internos son iguales entre s . 3. Los angulos alternos externos son iguales entre s . 4. Los angulos conjugados son suplementarios.

Figura 1: Angulos entre las rectas paralelas L1 y L2 .

Ejercicios
1. Tres angulos adyacentes forman un semiplano y tienen sus medidas proporcionales a los nmeros 5, 7 y 8. Hallar la medida del menor angulo. u 2. Demostrar que las bisectrices de dos ngulos suplementarios son perpendiculares. a

3. En la gura adjunta, L1 L3 L4 . Calcular x.

L2 y

4. Con ayuda de la gura 2, demuestre que: Si L1

L2 entonces = + .

Figura 2 5. En la gura 3, AB F G. Hallar el angulo x si el AM F = 90 y el M AB = 110.

Figura 3 6. Calcular el OP Q, si OP es bisectriz del angulo O, L1 L2 y P Q L1 . Ver gura 4.

Figura 4

7. En la gura 5, L1

L2 y L3

L4 , calcular .

Figura 5 8. En la gura 6, calcular x, si L1 L2 .

Figura 6

9. Calcular la medida del grco anexo, si las a rectas L1 y L2 son paralelas.

10. En la gura 7, L1

L2 y L3

L4 . Hallar el valor del ngulo . a

Figura 7 11. Sea AOB = 24, en la regin exterior a dicho angulo se traza el rayo OC. Hallar la o medida del angulo formado por las bisectrices de los ngulos AOC y BOC. a 12. Del grco 8, calcular y, cuando x tome su mximo valor entero. a a

Figura 8

2.

Tringulos: Teoremas Fundamentales. a

TEOREMAS FUNDAMENTALES EN TODO TRIANGULO.


Diremos que tres puntos que pertenecen a una misma recta son puntos colineales; de manera anloga, si tres rectas pasan por un mismo punto, sern llamadas rectas concurrentes. Si tomaa a mos al azar tres puntos en el plano, en muy raras ocasiones estos puntos estarn alineados,3 a y diremos entonces que son los vrtices de un tringulo; anlogamente sucede con las rectas, e a a tres rectas por lo general no concurren, y la gura geomtrica que stas denen es tambin un e e e tringulo.4 Una denicin completa para nuestros intereses es la siguiente: a o Denicin de Tringulo. Si A, B y C son tres puntos cualesquiera no colineales (Ver gura o a 9), entonces la reunin se los segmentos AB, BC y AC se llama tringulo ABC y se denota o a por ABC. Los puntos A, B y C se llaman vrtices y los segmentos AB, BC y AC se llaman e lados. Simblicamente: ABC = AB BC AC. Todo tringulo ABC determina tres ngulos o a a internos o interiores: ABC, ACB y BAC, y se llamar ngulo externo o exterior, al a a angulo determinado por un lado y la prolongacin del lado adyacente, en la gura 9, , y o son ngulos exteriores. a

Figura 9: Elementos del Tringulo a Dado el ABC, se tiene que AB + BC + CA = p = 2s, donde p es llamado el per metro y s el semiper metro del tringulo. Para abreviar, suele asociarse a cada vrtice un lado opuesto, a e y viceversa, por ejemplo, el lado opuesto de A es BC, y es frecuente que se denote por a; anlogamente b = CA, c = AB. a Teorema 1: En todo tringulo, la medida de un ngulo exterior es igual a la suma de las a a medidas de dos angulos interiores del tringulo no adyacentes a l. a e La demostracin de este teorema se basa en las relaciones de ngulos entre paralelas; se deja o a al lector que haga la demostracin (Sugerencia: por un vrtice, trace una recta paralela al lado o e
En teor de probabilidades, la probabilidad que esto ocurra es cero! a El trmino ms riguroso para esta gura es triltero. En este caso, habr que hacer una consideracin: si e a a a o hay un par de rectas paralelas, el triltero denido ya no es normal segn nuestro sentido comn, sin embargo, a u u sigue siendo un triltero! a
4 3

opuesto) Corolario: En todo tringulo, la suma de las medidas de sus tres ngulos internos es igual a a a 180. Teorema 2: Desigualdad Triangular. En todo tringulo, la longitud de uno de sus lados a est comprendido entre la suma y la diferencia de los otros dos. a Sin ser muy rigurosos, suponga que dado el segmento AB se traza con centro en A una circunferencia de radio r1 , y con centro en B una circunferencia de radio r2 ; si AB < r1 + r2 , las circunferencias se cortarn en dos puntos, y cualquiera de ellos puede ser el vrtice C, a e as AB < BC + CA; en cambio, si AB = r1 + r2 o peor an, si AB > r1 + r2 , la construccin u o del ABC no es posible. La Desigualdad Triangular es un resultado fundamental, a partir de sta y de su modelo de e demostracin se generan los Criterios de Congruencia de Tringulos; a groso modo, si dadas o a ciertas condiciones, la construccin de una gura geomtrica (un tringulo en particular) queda o e a determinada de manera unica, entonces dos guras que reunen las mismas condiciones sern a llamadas guras congruentes. As si se tienen tres segmentos (cuyas longitudes cumplen la desigualdad triangular), dejando , uno jo y construyendo las circunferencias con centros en los extremos de este segmento y radios las longitudes de los otros segmentos, por construccin, slo ser posible obtener dos o o a tringulos (uno con cada punto de interseccin de las circunferencias), que son bsicamente el a o a mismo pero la orientacin de los angulos es contraria; as si se sabe que dos tringulos cumplen o , a tener lados respectivamente iguales, por construccin, deben de ser iguales. Este es el conocido o criterio LLL de congruencia de tringulos; ms adelante se detallarn el resto de criterios, pero a a a a partir de este probaremos el siguiente resultado: Teorema 3: En todo tringulo, se cumple que a lados iguales se oponen ngulos iguales, y a a viceversa. Suponga que ABC es tal que AB = AC, entonces, por criterio LLL, ABC es congruente al ACB (en ese orden, porque AB = AC, BC = CB y CA = BA), entonces, los angulos que se oponen a los ngulos iguales son iguales. Para el rec a proco necesitamos otro criterio de congruencia, por lo que la demostracin se dejar incompleta; retome esto en la seccin de o a o congruencia de tringulos. a Teorema 4: En todo tringulo se cumple que a mayor lado se opone mayor angulo y viceversa. a Este teorema se deja como ejercicio para el lector (Sugerencia: utilice el teorema anterior, tome el lado mayor y dena un punto adecuado que genere un tringulo con dos lados iguales.) a

CLASIFICACION DE TRIANGULOS.
1. Con relacin a sus lados: o 8

a) Escaleno: si sus tres lados no son congruentes. b) Issceles: si por lo menos dos de sus lados son congruentes. o c) Equiltero: si sus tres lados son congruentes (note un tringulo equiltero es tambin a a a e issceles, y que los tres ngulos internos son iguales entre s e iguales a 60) o a 2. Con relacin a sus ngulos internos: o a a) Acutngulo: si su ngulo mayor es agudo (note que entonces los tres angulos son a a agudos) b) Rectngulo: si su angulo mayor es angulo recto (note que el ngulo en cuestin es a a o unico y que los otros dos ngulos son agudos; as en un tringulo rectngulo, la a , a a hipotenusa es mayor a los catetos) c) Obtusngulo, si el angulo mayor es ngulo obtuso (note que el angulo en cuestin a a o es unico y que los otros son agudos; as en un tringulo obtusngulo, el lado que se , a a opone al angulo obtuso es el lado mayor)

L INEAS NOTABLES EN UN TRIANGULO.


1. Altura: Se llama altura de un tringulo al segmento que parte de uno de sus vrtices y a e llega en forma perpendicular al lado opuesto o a su prolongacin. o 2. Mediana: Se llama Mediana al segmento que une un vrtice con el punto medio del lado e opuesto. 3. Mediatriz: Se denomina mediatriz de un lado de un tringulo es la recta perpendicular a a dicho lado en su punto medio. 4. Una Bisectriz: La bisectriz es la recta que divide en dos angulos iguales a un angulo dado; en particular, es bisectriz interna si es la bisectriz de un angulo interno de un tringulo, y bisectriz externa si es la bisectriz de un angulo externo de un tringulo. a a

DISTANCIA DE UN PUNTO A UNA RECTA.


En la gura 10, sea P un punto exterior a una recta L, la longitud de la perpendicular P M a la recta L es la distancia del punto P a dicha recta. Esta perpendicular tiene la propiedad de ser unica y su longitud es la distancia m nima del punto a la recta (Prubelo utilizano el hecho e que la hipotenusa es mayor que los catetos). Los segmentos P A y P B no son perpendiculares a L y se llaman oblicuas.

TEOREMA DE PITAGORAS.
Abordamos el estudio de las Relaciones Mtricas, del cual solo realizaremos el anlisis del fae a moso Teorema de Pitgoras, cuyo enunciado es el siguiente: a Teorema: Pitgoras. En un tringulo rectngulo, el cuadrado de la hipotenusa es igual a la a a a suma de los cuadrados de los catetos.

Figura 10 Una demostracin de este teorema es debida a Thabit ibn Qurra (836-901), la cual consiste en o diseccionar la gura que se forma al construir dos cuadrados de lados respectivamente iguales a los catetos de un tringulo rectngulo, como se muestra en el grco 11. a a a

Figura 11 Rec proco del teorema de Pitgoras: Si en un tringulo el cuadrado de un lado es igual a a a la suma de los cuadrados de los otros dos lados, el tringulo es rectngulo.5 a a

Ejercicios.

1. En la gura adjunta ambos tringulos son a equilteros. Encuentre el valor de . a

Ver demostracin en la seccin de congruencia de tringulos. o o a

10

2. En la gura 12, calcular el x si el AOB = 100 y L1

L2 .

Figura 12 3. (*) En la gura 13, ABDE es un cuadrado y BCD es un tringulo issceles con BD = a o DC. Si ABC = 160, determinar la medida de AEC.

Figura 13

4. (*) (XV Competencia de Clubes Cabri Primera Ronda) En la gura adjunta, ABCD es un rectngulo tal que AB = 2BC. M es el punto a medio de AB y los tringulos AM E y M BF son equilteros. Si P a a es la interseccin de las rectas DE y CF , encuentre los angulos del o CDP .

5. Si AB y F G son rectas paralelas, el ABC = CDE = , el DEF = 150. Calcule . Figura 14

y el GF H =

6. Probar que una bisectriz exterior de un tringulo es paralela al lado opuesto si y slo si a o el tringulo es issceles. a o 11

Figura 14 7. (*) Hallar la suma de los angulos + + + en la gura 15.

Figura 15 8. Determine el valor de la suma A + B + I + H + F + G. Figura 16.

Figura 16

12

9. En el ABC el BAC = 36 y AC = AB. Probar que la bisectriz interior BD, D en AC, es congruente con el lado BC. 10. Sea ABC un tringulo rectngulo en B con AB = BC, se construye exteriormente el a a tringulo equiltero BCD. Encuentre el angulo DAB. a a 11. En el ABC, AB = AC y D un punto sobre la recta AC, tal que BC = BD = DA. Determine la medida del angulo ABD, si: a) D est entre A y C. a b) A est entre D y C. a 12. En un ABC, D es un punto sobre el lado AC tal que AB = AD. Si ABC ACB = 90, hallar el CBD. 13. En la gura 17, el ABC = ACE, DC = EC, Qu l e nea notable es AD del BCA?

Figura 17 14. Se tiene un tringulo issceles ABC, AB = BC en el cual se traza al altura AF tal que a o BF = 6 y F C = 2. Hallar AC. 15. Cul es el valor de b a en la gura 18? a

Figura 18

13

16. La hipotenusa BC de un tringulo rectngulo ABC se divide en 4 segmentos congruentes a a por los puntos G, E y H. Si BC = 20, encuentra la suma de los cuadrados de las longitudes de los segmentos AG, AE y AH. Figura 19.

Figura 19 17. (*) Dado un cuadrado ABCD, se construyen los tringulos equilteros ABP (exteriora a mente) y ADQ (interiormente). Probar que C, P y Q estn alineados. a 18. (*) Sea ABC un tringulo rectngulo con CAB = 90. D es un punto sobre la prolona a gacin de BC tal que BD = BA. E es un punto en el mismo semiplano que A respecto o de BC, tal que CE BC y adems CE = CA. Mostrar que A, D y E estn alineados. a a 19. El cuadriltero ABCD mostrado en la gura 20 cumple que AB CD y BC DA.6 a Sobre las prolongaciones de AB y AD se construyen puntos E y F tales que BC = BE y DC = DF . Demuestre que C, E y F estn alinedos. a

Figura 20

20. (*) En la gura adjunta, AB = BC = CD = DE = EF = F G = GA. Calcule la medida del DAE.

El cuadriltero ABCD es un paralelogramo. a

14

21. (*) (XXVIII Olimpiada Brasilea de Matemtica) En la gura 21, AB = AC, AM = AN n a y CAM = 30, encuentre el valor del BM N .

Figura 21 22. Los lados de un tringulo issceles son 12 y 5 metros, cul es su per a o a metro? 23. Muestre que los lados de un tringulo cumplen que |a b| < c y que c < a
a+b+c . 2

24. Muestre que es posible construir un tringulo con segmentos de longitudes a, b, c si y slo a o existen nmeros positivos x, y, z tales que: a = x + y, b = y + z, c = z + x. u Problemas de Refuerzo. 25. (*) (Etapa seminal Estatal de XXII Olimpiada Mexicana de Matemticas) En la gura a 22 se muestra un hexgono regular ABCDEF de lado 1. Los arcos del c a rculo que estn a dibujados tienen centro en cada vrtice del hexgono y radio igual a la distancia al vrtice e a e opuesto. P , Q, R, S, T y U son los puntos de corte de estos arcos. Cunto mide cada a lado del hexgono P QRST U ? a

Figura 22

15

3.

Congruencia de Tringulos. a

CRITERIOS DE CONGRUENCIA.
Denicin de Congruencia de tringulos. El ABC es congruente al A B C si: AB = o a A B , AC = A C , BC = B C , ABC = A B C , ACB = A C B y BAC = B A C . Simblicamente: ABC = A B C . Vase gura 23. o e

Figura 23: Denicin de Igualdad de Tringulos. o a La denicin anterior establece que dos tringulos son congruentes si tanto los lados como o a los angulos se presentan en pares respectivos congruentes. Esto, segn la visin de Euclides, u o signica que un tringulo es posible superponerlo sobre el otro (se puede desplazar, girar o a reejar) y coincidir de manera perfecta. Sin embargo, es importante mencionar que en muy a raras ocasiones se tendr a disposicin tanta informacin, de all la importancia de los criterios a o o de congruencia, que establecen los requisitos m nimos para garantizar que dos tringulos son a congruentes. El siguiente es el primero de los tres criterios de congruencia de tringulos, y se denomina a criterio de LADO-ANGULO-LADO, en s mbolos: L-A-L. Criterio L-A-L. Si los tringulos ABC y A B C presentan las congruencias: AB = A B , a AC = A C y BAC = B A C , entonces ABC = A B C .

Figura 24: Criterio LAL Segn el criterio L-A-L, dos tringulos son congruentes si en uno de ellos existen dos lados y el u a angulo (comprendido entre dichos lados), respectivamente congruentes a dos lados y el angulo (comprendido entre dichos lados), en el otro tringulo. a Criterio A-L-A. Sean ABC y A B C dos tringulos tales que: AC = A C , BCA = B C A a y BAC = B A C , entonces ABC = A B C . 16

Figura 25: Criterio ALA. Criterio L-L-L. Si un tringulo tiene sus tres lados respectivamente congruentes a los tres a lados de otro tringulo, entonces estos dos tringulos son congruentes. a a

Figura 26: Criterio LLL. Ahora demostraremos el Recproco del Teorema de Pitgoras. a Demostracin: Sea ABC un tringulo talque BC 2 = AB 2 + AC 2 , por construccin sea el o a o A B C rectngulo en A tal que A B = AB y A C = AC, entonces por el teorema de a Pitgoras B C 2 = A B 2 + A C 2 , as que B C 2 = BC 2 , de donde B C = BC y por el criterio a LLL, se deduce que el A B C = ABC, por lo tanto el BAC = B A C = 90.

TEOREMA DE LA BASE MEDIA


En todo tringulo, el segmento que une los puntos medios de dos lados es paralelo al tercer a lado e igual a su mitad. En la gura 27, M N es el segmento que une los puntos medios de los lados AB y BC del ABC, AC a este segmento se le llama BASE MEDIA DEL TRIANGULO. Se verica que M N = y 2 que M N AC. Demostracin: o 1. Prolongar el segmento M N hasta el punto P tal que M N = N P . 2. Los tringulos M N B y P N C son congruentes, ya que BN = N C, M N = N P y el a P N C = BN M , por consiguiente, el N CP = M BN , por lo tanto, CP M B (Por angulos alternos internos iguales). Adems, P C = M B = M A; con lo cual se tiene que: a M A = P C. 17

Figura 27: Teorema de La Base Media.

Figura 28: Menor Media en un Tringulo Rectngulo. a a 3. Uniendo el punto A con el punto P se forman los tringulos congruentes AM P y ACP a (por L A L) ya que M A = P C, AP = AP , M AP = AP C (por angulos alternos internos entre las paralelas M A y P C). Luego, M P = AC, entonces N P = 1 M P = 1 AC. 2 2 Adems, P AC = M P A, de donde M P AC o que M N AC. a Corolario: Menor mediana de un tringulo rectngulo. En todo tringulo rectngulo, a a a a la mediana relativa a la hipotenusa es la mitad de la longitud de la hipotenusa y es la menor de las tres medianas del tringulo. a Demostracin: En la gura 28, BM es la mediana relativa a la hipotenusa AC del ABC, o a probaremos que BM = AC ; (con lo cual se tendr que BM = AM = M C). Si por M se traza 2 una paralela al lado AB, que corte al lado BC en N , entonces N es el punto medio de BC y el M N C = 90, los tringulos BN M y CN M son congruentes por el criterio L-A-L, luego a M B = M C = AM . Probar que BM es la menor mediana (Ejercicio).

18

Ejercicios.

1. (*) En la gura adjunta, ABC es un tringulo equiltero a a y CDEF es un cuadrado. Se construye un punto G tal que CF = CG y adems CF G = 15. Probar que a AGC = BDC.

2. Dado un tringulo equiltero ABC, se construye un tringulo equiltero DEF cuyos a a a a vrtices estn sobre los lados del ABC, tal como muestra la gura 29. Demuestre que e a los tringulos ADF , BED, CF E son todos congruentes entre si. a

Figura 29 3. ABCD es un cuadrado, E, F , G y H son puntos sobre los lados AB, BC, CD, DA, respectivamente, tal que EF GH tambin es cuadrado. Demuestre que los tringulos AEH, e a BF E, CGF , DHG son todos congruentes entre si. Figura 30.

Figura 30

19

4. ABCDE y F GHIJ son pentgonos regulares (Vease gura 31). Demuestre que los a tringulos AF J, BGF , CHG, DIH, EJI son todos congruentes entre si. a

Figura 31 5. Si AB CD y AB = CD entonces, AD = BC y AD BC 7 .

6. Demuestre que dos tringulos desplazados a son congruentes. Sugencia: Utilice el problema anterior.

7. Demuestre que dos tringulos rotados son congruena tes.

El cuadriltero ABCD se denomina paralelogramo. a

20

8. Demuestre que dos tringulos reejados con respecto a un punto a

son congruentes.

9. Demuestre que dos tringulos reejados con respecto a a una recta son congruentes.

Importante: Las traslaciones, rotaciones y reexiones no cambian el tamao ni la forma n de un tringulo. a

10. (*) En la gura adjunta, ABCD un cuadrado y EF GH. Demuestre que que EF = GH.

11. Dos cuadrados ABCD y EHGF , ambos de lado l, estn colocados en manera tal que un vrtice de uno a e est en el centro del otro (como en la gura anexa). a l2 Demuestre que el rea del cuadriltero EJBK es a a 4 y por ende no depende de la posicin de J (o K). o

La reexin con respecto a un punto es equivalente a una rotacin de 180 o o

21

12. En un ABC el B = 2C, la mediatriz del lado AC corta en F al lado BC. Hallar AB, si BF = 3 y F C = 9. 13. (*) (Examen nal de XVI Olimpiada mexicana de Matemtica) Los angulos de un tringua a lo ABC estn en progresin aritmtica (B A = C B = ), D, E, y F son los a o e puntos medios de los lados BC, CA y AB, respectivamente. Llamamos H al pie de la altura trazada desde C (que cae entre B y F ) y G a la interseccin entre DH y EF . o Cunto vale F GH? a 14. En la gura 32, AC = 12 AF = 4 y BAF = 30. Hallar BF si AG = GC.

Figura 32 15. En la gura 33, AG = GC, el AF G = 20. Hallar el F AC, si AC = 2BF .

Figura 33 16. (*) Sea ABCD un cuadrado. Se construyen tringulos equilteros ADP y ABQ como se a a muestra en la gura 34. Sea M la interseccin de CQ con AD y N la interseccin de CP o o con AB. Demuestre que CM N es un tringulo equiltero. a a

22

Figura 34 Problemas de Refuerzo. 17. En la gura 35, ABC, CDE y EF A son tringulos issceles, con el ABC = CDE = a o EF A = 120. Probar que el BDF es equiltero. a

Figura 35 18. (*) ABC es un tringulo issceles con ABC = ACB = 80. D es un punto en AC a o tal que ABD = 10. Demuestre que AD = BC.

23

4.

Cuadrilteros: Clasicacin y Propiedades. a o

CLASIFICACION.
Los cuadrilteros pueden clasicarse de acuerdo a sus diagonales de la siguiente forma: a Cuadriltero Convexo: Es un cuadriltero con las dos diagonales en su interior. a a Cuadriltero Entrante: Es un cuadriltero con una diagonal en el interior y otra en el extea a rior. Cuadriltero Cruzado Es un cuadriltero con las diagonales en su exterior.9 a a Es muy frecuente que se considere que un cuadriltero es convexo, a menos que se especique lo a contrario. Esto es as porque muchos resultados son ms claros en un cuadriltero convexo, sin a a embargo, es importante darse cuenta que existen teoremas que no se cumplen para cualquier tipo de cuadrilteros, por ejemplo: a Teorema: La suma de los ngulos internos de un cuadriltero no cruzado es 360. a a La demostracin de este resultado se basa en la diseccin del cuadriltero en dos tringuo o a a los cuyos angulos internos conforman los ngulos internos del cuadriltero, sin embargo, estas a a condiciones no pueden lograrse en un cuadriltero cruzado; de hecho, la suma de los angulos a internos puede hacerse arbitrariamente pequea cuando el cuadriltero es cruzado. n a Tambin hay otras clasicaciones de cuadrilteros de acuerdo a sus lados y ngulos. e a a Cuadriltero Equingulo: un cuadriltero (convexo) es equingulo si todos sus ngulos intera a a a a nos son iguales; dado el teorema anterior, los ngulos son iguales a 90, por ello este cuadriltero a a es llamado rectngulo. a Cuadriltero Equiltero: un cuadriltero (convexo) es equiltero si todos sus lados son iguaa a a a les. A este cuadirtero tambin se le conoce como rombo. a e Cuadrado: es un cuadriltero que es equingulo y equiltero. a a a Paralelogramo: es un cuadriltero con los lados opuestos paralelos. a Trapecio: es un cuadriltero con un par de lados opuestos paralelos.10 a
Tanto los cuadrilteros convexos como los entrantes son cuadrilteros simples, que son los cuadrilteros a a a cuyos lados no se cortan salvo en los extrenos; en contraposicin, los cuadrilteros cruzados no son simples. o a 10 Note que un paralelogramo es tambin un trapecio. e
9

24

PARALELOGRAMOS
Dado el paralogramo ABCD, por propiedades de angulos entre paralelas es posible probar el siguiente resultado: Teorema: Los angulos opuestos son iguales y los ngulos consecutivos son suplementarios: a ABC = CDA = y BCD = DAB = 180 . Por otra parte, por criterio ALA, i.e. ABC CDA; esto implica que AB = CD y BC = DA,

Teorema: Los lados opuestos de un paralogramos son iguales. A partir de esto, si M es la interseccin de AC con BD, por criterio ALA, o por lo que AM = CM y BM = DM , i.e. Teorema: Las diagonales de un paralelogramo se bisecan. Adems, se cumple un resultado sosticado y muy importante: a Teorema: Ley del Paralelogramo. Si ABCD es un paralelogramo entonces el doble de la suma de los cuadrados de los lados es igual a la suma de los cuadrados de las diagonales, es decir 2 AB 2 + BC 2 = AC 2 + BD2 Demostracin: Aplicando la Ley del Coseno a o ABC y ABD se tiene ABM CDM ,

AC 2 = AB 2 + BC 2 AB BC cos DB 2 = AB 2 + AD2 AB AD cos(180 ) AC 2 + DB 2 = 2 AB 2 + BC 2 AB BC (cos + cos(180 )) y dado que cos = cos(180 ) el resultado se sigue inmediatamente.

RECTANGULOS
En primer lugar, es importante notar que todo rectngulo es paralelogramo (por angulos ena tre paralelas), por lo que todos los resultados probados anteriormente son heredados a todo rectngulo; pero los rectngulos tienen propiedades adicionales: a a Observe que por criterio LAL, ABC ABD, por lo que AC = BD y entonces Teorema: Las diagonales de un paralelogramo son iguales; adems, el punto de interseccin a o de estas equidista de los cuatro vrtices y por tanto es el centro de una circunferencia que pasa e por todos los vrtices. e Por otra parte, observe que si se aplica la ley del paralelogramo a un rectngulo se obtiene el a Teorema de Pitgoras. a 25

ROMBOS
Dado un rombo ABCD, por criterio LLL, ABC CDA, y por lo tanto BAC = DAC y BCA = DAC, lo cual implica BC AD y AB CD, i.e., todo rombo ABCD es un paralelogramo. Adems, por las mismas congruencias se tiene a Teorema: Las diagonales de un rombo cumplen ser una mediatriz de la otra. Teorema: Las diagonales de un rombo bisecan a los angulos interiores del rombo; esto implica que el punto de corte de las diagonales equidista de los cuatro lados del rombo y es el centro de una circunferencia tangente a estos.

TRAPECIOS
Dado el trapecio ABCD (con AB CD), se construyen los puntos medios de BC y DA, M y N , respectivamente. Si el cuadriltero M N AB se rota con centro en M y angulo 180 se genera a un cuadriltero M N A C; observe que N D = N A y N D N A , por lo que DN N A es un a paralelogramo y N N = DA 2M N = DC + CA 2M N = DC + AB AB + CD MN = 2 El segmento M N es llamado base media del trapecio, y por lo recin demostrado se tiene e Teorema: La base media de un trapecio es igual a la semisuma de las bases. Por otra parte, hay ciertos trapecios que reciben nombres particulares; el trapecio rectngulo es a aquel que las bases son perpendiculares a alguno de los otros lados; y por otra parte, el trapecio issceles es aquel que los lados (distintos de las bases) tienen igual longitud. 11 o

Ejercicios
1. Dado el trapecio ABCD con AB CD, demuestre que la bisectriz interior del A es paralela a la bisectriz exterior del D. 2. A un rombo ABCD se le construyen exteriormente los cuadrados ABEF y BCGH. Demuestre que ABD = EBH. 3. (*) Sea ABCD un paralelogramo. Se construyen tringulos equilteros exteriores a a y ADQ. Demuestre que el BP Q es equiltero. a CDP

4. Demuestre que las bisectrices interiores de un paralelogramo forman un rectngulo (qu sua e cede si el paralelogramo es adems rombo?). a
Los trapecios issceles son muy importantes cuando se estudian los ngulos en la circunferencia; resulta que o a un trapecio es issceles si y slo si los cuatro vrtices se ubican sobre una misma circunferencia. o o e
11

26

5. Demuestre que las bisectrices exteriores de un paralelogramo forman un rectngulo. a 6. Sea ABCD un paralelogramo. La bisectriz interna del CDA corta a BA en M , y la bisectriz interna del BAD corta a CD en N . Demuestre que ADN M es un rombo. 7. Demuestre que si por el punto de interseccin de las diagonales de un rombo se trao zan perpendiculares a los lados del rombo, entonces los puntos de interseccin de dichas o perpendiculares con los lados del rombo forman un rectngulo. a 8. Demuestre que las bisectrices de los ngulos denidos por las diagonales de un rombo, a cortan a los lados del rombo en cuatro puntos que forman un cuadrado. 9. En un ABC sea G la interseccin de las medianas BB y CC . Sean B , C las reexiones o de G respectivas a los puntos B y C . a) Demuestre que AGCB y AGBC son paralelogramos. b) A partir de lo anterior, demuestre que BCB C tambin es paralelogramo. e c) Demuestre que A pertenece a la recta AG, y concluya que las tres medianas de un tringulo concurren en el punto G, llamado el centroide del ABC. a d) Demuestre que CG = 2GC ; relaciones similares se cumplen para las otras dos medianas. 10. Teorema de Varignon: Dado un cuadriltero ABCD (no necesariamente convexo), se a construyen los puntos medios L, M , N , O, P , Q, de los segmentos de recta AB, BC, CD, DA, BD, AC, respectivamente. Figura 36. a) Demuestre que LM N O, LP N Q, OP M Q, son paralelogramos. b) Demuestre que LN , OM , P Q concurren en un punto, llamado el centroide del cuadriltero ABCD. a c) Demuestre que el per metro de LM N O es igual a AC + BD; resultados similares se cumplen para los otros paralelogramos.

Figura 36: Teorema de Varignon

27

11. Sea ABCD un paralelogramo tal que existe un punto E sobre el lado AB que cumple CED = 90. Sean M y N los pies de las perpendiculares trazadas desde A y B hacia DE y CE, respectivamente. Demuestre que AC, BD y M N concurren. 12. (*) (Hctor Alberti) Sea ABCD un cuadrado. Se construyen los tringulos equilteros e a a BDA , ACB , BDC y ACD . Demuestre que el A B C D es tambin un cuadrado. e 13. (*) (II Olimpiada Matemtica del Cono Sur) En la gura 37 ABCD y AECF son paraa lelogramos. Demuestre que BEDF es paralelogramo.

Figura 37 Problemas de Refuerzo. 14. (*) ABCD es un cuadriltero convexo y O es un punto en su interior. Sean P , Q, R, S, a los puntos medios de los lados AB, BC, CD, DA, respectivamente. Por P se traza una paralela a OR, por Q se traza una paralela a OS, por R se traza una paralela a OP , y por S se traza una paralela a OQ. Demuestre que estas cuatro rectas concurren. 15. (*) Un trapecio issceles tiene diagonales perpendiculares y su area es 2010, determine su o altura. 16. (*) (IX Competencia de Clubes Cabri, Segunda Ronda) Sea ABCDEF un hexgono a regular cuyo centro es O. Se construyen los cuadrados F SOP y ORCQ. Demuestre que AP QB y SEDR son rectngulos. Figura 38. a 17. (*) Sobre los lados del ABC se trazan exteriormente los cuadrados ABP Q, CARS y BCT U . Luego se trazan los paralelogramos AQA R, CSC T y BU B P . a) Sean A , B , C los centros de los cuadrados BCT U , CARS, ABP Q, respectivamente. Demuestre que estos centros estn sobre los lados del A B C . a b) Demuestre que AA , BB , CC concurren.

28

Figura 38 18. (*) Se dibujan cuadrados exteriores a los lados de un paralelogramo, demuestre que: a) El cuadriltero determinado por los centros de esos cuadrados es un cuadrado. a b) Las diagonales de ese cuadrado son concurrentes con las del paralelogramo. 19. (*) Dado un ABC, se construyen exteriormente los tringulos rectngulo issceles a a o ACP y BCQ, con AC y BC como hipotenusas. Si M es el punto medio de AB, demuestre que el M P Q tambin es un tringulo rectngulo issceles. e a a o

29

5.

Angulos en la Circunferencia.

LA CIRCUNFERENCIA Y SUS ELEMENTOS


Una circunferencia es el lugar geomtrico de puntos que equidistan de un punto dado, llamado e el centro de la circunferencia; la distancia de cada punto de la circunferencia al centro es el radio. Por otra parte, todos los puntos que estn a una distancia del centro menor o igual al radio a forman el c rculo; estos puntos quedan al interior o sobre la circunferencia. Si A y B son dos puntos de una circunferencia, el segmento de recta AB dene una cuerda; en particular, si el centro de la circunferencia pertenece a la cuerda, sta es llamada dimetro. Es e a importante mencionar que para cada punto de la circunferencia existe exactamente un punto diametralmente opuesto. En la gura 39, se tiene una circunferencia de centro O y radio r = OA = OB = OA ; AB y AA son cuerdas, pero AA es tambin dimetro, i.e, A es diametralmente opuesto a A y e a viceversa. Observe que por la desigualdad triangular aplicada al tringulo issceles AOB a o

Figura 39

AB < AO + BO = r+r = AA Si A es un punto jo, esta desigualdad es vlida para cualquier punto B sobre la circunferencia a (excepto cuando B = A lo cual implica AB = AA ). Esto quiere decir que el dimetro es la a mayor de todas las cuerdas. A las porciones de circunferencia que quedan entre dos puntos ubicados en la circunferencia, se les llama arcos de circunferencia; note que dos puntos sobre una circunferencia denen dos arcos de circunferencia. Tambin, si un angulo tiene vrtice sobre el centro de la circunferencia y e e est formado por dos radios, ser llamado ngulo central ; de nuevo, AOB hace referencia a dos a a a angulos, cuya suma es 360, y subtienden respectivamente a uno de los arcos AB. Finalmente, si un angulo tiene el vrtice sobre la circunferencia y est formado por dos cuerdas, ser llama e a a do ngulo inscrito; en la gura anterior, AA B es un angulo inscrito que subtiende al arco AB. a

30

Teorema: El ngulo central es el doble del ngulo inscrito que subtiende el mismo arco. a a Demostracin: Considere la gura 40, se demostrar que AOB = 2AP B en los tres casos o a mostrados. En la circunferencia de la izquierda, sea P el punto diametralmente opuesto a P ; observe que AP O y BP O son tringulos issceles, y por el teorema del ngulo externo se a o a tiene AOB = = = = = AOP + BOP (AP O + OAP ) + (BP O + OBP ) 2AP O + 2BP O 2 (AP O + BP O) 2AP B

Figura 40 El caso de la circunferencia del medio es ms sencillo y se deja como ejercicio para el lector. a Para la circunferencia de la derecha, el trabajo es anlogo y slo cambia en un pequeo arreglo a o n algebraico AOB = = = = = BOP AOP (BP O + OBP ) (AP O + OAP ) 2BP O 2AP O 2 (BP O AP O) 2AP B

Corolario: Todos los angulos inscritos que subtienden el mismo arco son iguales (Ver gura 41). En particular, los angulos internos son iguales a 90 si subtienden a una semicircunferencia. Demostracin: Todos los angulos mostrados en la gura 41 son iguales a la mitad del AOB, o y por tanto, son iguales entre s En particular, si AB fuera un dimetro, AOB = 180 y por . a tanto AP B = 90. 12 Hay un par de angulos ms que son importantes: Si un punto P es interno a la circunferencia, a el ngulo de vrtice P formado por dos cuerdas que pasan por P se llama ngulo interior. De a e a
12

Observe que en cualquier tringulo rectngulo, el punto medio de la hipotenusa equidista de los tres vrtices. a a e

31

Figura 41 forma similar, si P es exterior y dos cuerdas de la circunferencia (al prolongarse) pasan por P , el ngulo con vrtice P es llamado angulo exterior. a e Dejamos como ejercicio demostrar el siguiente teorema: Teorema: Los ngulos interior y exterior mostrados en la gura 42 cumplen las frmulas a o siguientes: AQC = BOD + AOC 2 BOD AOC AP C = 2

Figura 42

CUADRILATEROS C ICLICOS
Ahora suponga que sobre una circunferencia se ubican cuatro puntos A, B, C, D, como se muestra en la gura 43. Al cuadriltero ABCD se le llama cuadriltero cclico o concclico. a a Observe que ABC + CDA = + = 180 . 2 2 32

Figura 43 Y anlogamente DAB +BCD = 180. Esto signica que si ABCD es un cuadriltero c a a clico y convexo, entonces los ngulos opuestos son suplementarios. Tambin, es posible demostrar por a e contradiccin el rec o proco de este resultado: si suponemos que ABCD es tal que B+D = 180 pero no es c clico, se dene el punto D como la otra interseccin de AD con el circunc o rculo del ABC, y como ABCD es c clico (por construccin) entonces B + D = 180, luego, o D = D , lo cual implica la contradiccin CD CD (rectas paralelas que se cortan en C). o As se ha demostrado el siguiente teorema: , Teorema: El cuadriltero convexo ABCD es un cuadriltero c a a clico si y slo si o A + C = 180 = B + D Tambin, otro criterio muy util y cuya demostracin tambin se basa en el corolario anterior es e o e Teorema: El cuadriltero convexo ABCD es un cuadriltero c a a clico si y slo si se cumple o alguna de las siguientes igualdades ABD BCA BAC CAD = = = = ACD BDA BDC CBD

Es importante recalcar que NO todo cuadriltero puede ser inscrito en una circunferencia; por a ejemplo, un paralelogramo no ser c a clico a menos que sea rectngulo. a

RECTAS Y CIRCUNFERENCIAS TANGENTES A UNA CIRCUNFERENCIA


Dada una circunferencia, una recta puede ser tangente o secante a la circunferencia, dependiendo si la corta en uno o dos puntos, respectivamente; en cualquier otro caso, se dice que la

33

recta no corta a la circunferencia.13 Sea l una recta secante a la circunferencia que corta a la circunferencia en A y B (A = B); como el AOB es issceles, OAB < 90. Rec o procamente, si por A se traza una recta l tal que uno de los ngulos que forma con OA es menor que 90, se puede construir un punto B a sobre l tal que OAB = ABO < 90 y A = B (basta proyectar O sobre l y luego reejar A con respecto a este punto, el resultante es el punto B); entonces el AOB es issceles, por lo o que OA = r = OB, i.e. B pertenece a la circunferencia y por tanto l corta a la circunferencia en dos puntos distintos. As Teorema: Una recta l corta a una circunferencia de centro O en dos puntos distintos A y B si y slo si un angulo entre l y OA es agudo. o Corolario: Si l es una recta tangente en A a una circunferencia de centro O, ninguno de los angulos entre l y OA puede ser agudo, y por tanto l OA. A partir de este resultado se prueban otros resultados muy conocidos y utiles, que dejamos de ejercicios para el lector. Teorema: Dado un punto P externo a una circunferencia de centro O, si P A y P B son segmentos tangentes a la circunferencia en A y B, respectivamente, entonces el cuadriltero P AOB es a c clico y bissceles. o Corolario: Dado un punto P externo a una circunferencia de centro O, la circunferencia de dimetro P O corta a la circunferencia dada en dos puntos A y B tales que P A y P B son rectas a tangentes. Denicin: El angulo semi-inscrito en una circunferencia es aquel que se forma con una cuerda o y la recta tangente en alguno de los extremos de la cuerda. Teorema: La media del angulo semi-inscrito denido por la cuerda AB es igual a la medida de un angulo inscrito que subtiende al arco AB. Demostracin: Considere la gura 44. Como AP BO es c o clico, entonces P AB = P OB; adems, como P O es la mediatiz de AB, P OB = P OA, por lo que a P AB = AOB = AQB 2

Por otra parte, dada una circunferencia, otra circunferencia puede ser secante o tangente a la primera, dependiendo si la corta en uno o dos puntos, respectivamente; en cualquier otro caso se dice que las circunferencias no se cortan.14
Cuando la recta es tangente a la circunferencia puede considerarse como un caso muy peculiar en el cual los dos puntos de corte coinciden. 14 Tambin ac puede considerarse a las circunferencias tangentes como un caso especial de circunferencias e a secantes en el cual los puntos de corte coinciden.
13

34

Figura 44 Adems, dos circunferencias pueden posicionarse una dentro de la otra, y claramente, la cira cunferencia de radio mayor es la externa mientras que otra es la interna; particularmente, si las circunferencias tienen el mismo centro se llaman concntricas. Finalmente, combinando estas e denciones se tienen las circunferencias tangentes exteriormente y las tangentes interiormente. Teorema: Dadas dos circunferencias de centros O1 y O2 que se cortan en dos puntos distintos A y B, se cumple que O1 O2 AB. Teorema: Si dos circunferencias de centros O1 y O2 son tangentes en A, se cumple que O1 , A y O2 estn alineados. a Teorema: a) Dos circunferencias, una dentro de la otra, no tienen rectas tangentes en comn. u b) Dos circunferencias tangentes interiormente tienen una recta tangente comn. u c) Dos circunferencias secantes (en dos puntos distintos) tienen dos rectas tangentes en comn. u d) Dos circunferencias tangentes exteriormente tienen tres rectas tangentes en comn. u e) Dos circunferencias no secantes y tal que ninguna contiene a la otra, tienen cuatro rectas tangentes en comn. u

Ejercicios

1. Si el M P Q = 20, determine el valor del QON en la gura adjunta.

35

2. Dado un angulo inscrito BAC, y su ngulo central BOC, se sabe que BAC + BOC = a 180. Calcular el OBC. 3. En la gura 45, BCDO es un rombo. Determine el valor del angulo y la medida de las diagonales de BCDO si el radio de la circunferencia mide 6.

Figura 45 4. Un cuadriltero c a clico ABCD satisface ABC = 2CDA = . Calcule . 5. En la gura 46, P R es una tangente comn. Calcule el valor del P QR. u

Figura 46

6. En la gura adjunta, el AF E = 100 y el BCD = 150. Calcule el AGB.

7. Dado un angulo AOB, se trazan dos rectas l OA y m OB. Si P es el punto de corte de l y m, demuestre que A, B, O, P se ubican sobre una misma circunferencia. 36

8. Las bisectrices BP y CQ del entonces P QI es issceles. o

ABC se cortan en I. Demuestre que si BAC = 60

9. En la gura 47 se ha tomado un punto C sobre la circunferencia; AC y BC cortan a la segunda circunferencia en D y E respectivamente. Probar que OC DE.

Figura 47 10. (*) Dada la gura 48, demuestre que AB AB.

Figura 48 11. En la gura 49 CR es una recta tangente en C, demuestre que AB CR.

Figura 49

37

12. Dos circunferencias 1 y 2 son tangentes (interior o exteriormente) en P (Ver gura 50). Dos rectas que pasan por P cortan a 1 y 2 en A y C, y en B y D, respectivamente. Demuestre que AB CD.

Figura 50

13. (*) Dos circunferencias de centros O1 y O2 son tangentes (interna o externamente) en un punto P ; por este punto se traza una recta que corta nuevamente a la circunferencias en A y B, respectivamente. Demuestre que AO1 BO2 . 14. Dos circunferencias son tangentes externamente en el punto A. Una tangente exterior comn toca a una circunferencia en B y a la otra en C. Demostrar que BAC = 90. u 15. En la gura 51, DE es tangente en D, y C es el punto medio del arco AD. Encuentre el valor del ngulo seminscrito ADE. a

Figura 51

38

16. Determine el valor del DCF , sabiendo BE es tangente en el punto D a la circunferencia de centro O. Ver Figura 52.

Figura 52 17. Si el AEB = 30, ADE = 20 y ACE = 35, calcule el AF B. Vase gura 53. e

Figura 53 18. Dada una circunferencia de dimetro BC, se toma un punto P en la prolongacin de BC, a o y se traza la tangente AP . Si AP = AB y O es el centro de la circunferencia, demuestre que el AOC es equiltero. a 19. (*) Dadas dos circunferencias una fuera de la otra, demuestre que las tangentes comunes externas forman segmentos iguales; anlogamente, las tangentes comunes internas forman a segmentos iguales. 20. (*) Teorema de Pithot. Demuestre que en todo cuadriltero inscribible, la suma de a lados opuestos es igual. 21. (*) Teorema de Steiner. En todo cuadriltero exinscrito a una circunferencia, la difea rencia de las longitudes de lados opuestos es igual. 39

22. Demuestre que las mediatrices de un cuadriltero son concurrentes si y slo si es c a o clico. 23. Demuestre que el cuadriltero convexo ABCD es inscribible si y slo si los inc a o rculos respectivos del ABC y CDA son tangentes. 24. Demuestre que las bisectrices internas de un cuadriltero son concurrentes si y slo si es a o inscribible. 25. Demuestre que todo rombo es inscribible. 26. En la gura 54, AB es una cuerda y por D se traza una recta tangente a la circunferencia paralela a AB. Demuestre que CD es bisectriz del ACB.

Figura 54 27. Determine las medidas de ACB y ACO de la gura 55.

Figura 55

28. Cuatro cilindros de dimetro 1 estn pegados apretadamente por a a una cuerda muy na, como en la gura adjunta. Demostrar que la cuerda tine longitud 4 + . Demostrar tambin que el rea some a breada entre los cilindros es 1 4 .

29. En la gura 56, ABCD es un trapecio issceles con AB CD y DA = BC = 2; tomando o DA y BC como dimetros, se construyen dos circunferencias tangentes. Si DC = 3AB, a calcule el area del trapecio. 40

Figura 56 30. La gura 57 est formada por un paralelogramo y dos circunferencia tangentes entre s y a tangentes a tres lados del paralelogramo. Sabiendo que el radio de las mismas mide la cuarta parte del lado menor del paralelogramo, calcule la razn entre el lado mayor del o paralelogramo y el radio de las circunferencias.

Figura 57 31. En la gura 58, ABCDEF es un hexgono regular y las circunferencias de centro en los a vrtices son tangentes dos a dos. Si las circunferencias sobre los vrtices B, D, F son e e iguales, demuestre que las circunferencias restantes son iguales.

Figura 58 32. Alrededor de una circunferencia se construyen diez circunferencias tangentes a la original y tangentes entre s (Vase gura 59). Demuestre que la suma de las reas de las diez e a circunferencias es el doble del rea de la circunferencia mayor. a 41

Figura 59 33. (*) Teorema de Miquel: Dado un ABC, sean X, Y , Z puntos sobre AB, BC, CA, respectivamente. Demuestre que los circunc rculos de AXZ, BY X, CZY tienen un punto en comn M . u 34. (X OMCC - P2, Aarn) Sea ABCD un cuadriltero conc o a clico con dimetro AC, y sea a O el centro de su circunferencia. Se construyen los paralelogramos DAOE y BCOF . Demuestre que si E y F estn sobre la circunferencia entonces ABCD es rectngulo. a a 35. (*) Sea ABC un tringulo, y sean L y N las intersecciones de la bisectriz del angulo A a con el lado BC y el circunc rculo de ABC respectivamente. Construimos la interseccin o M del circunc rculo de ABL con el segmento AC. Prueba que los tringulos BM N y a BM C tienen la misma area. 36. (*) Sea AB el dimetro de una semicircunferencia. Se colocan los puntos M y K sobre a la semicircunferencia y sobre AB, respectivamente.15 Sea P el centro de la circunferencia que pasa por A, K y M ; sea Q el centro de la circunferencia que pasa por B, K y M . Demuestre que M P KQ es conc clico. 37. (*) Las circunferencias 1 y 2 se cortan en los puntos A y B. Por el punto A se traza una recta que corta nuevamente a las circunferencias 1 y 2 en los puntos C y D, respectivamente. Por los puntos C y D se trazan tangentes a las circunferencias, las cuales se cortan en el punto M . Demuestra que M CBD es c clico. 38. (*) El ABC cumple que A = 90 y AB = AC. Se toma un punto E del segmento AB, se construye interiormente un tringulo equiltero AEF . EF corta BC en I, y se a a construye exteriormente un tringulo equiltero BIJ. Encuentre EJB. a a 39. (*) En la gura 60, se sabe que AO1 B AO2 B = 70 y adems la tangente EB forma a el tringulo issceles ABE, con AB = AE. Encuentre EBC. a o
15

M y K son distintos de A y B.

42

Figura 60 40. (*) Dos circunferencias 1 y 2 se cortan en A y B. Una recta por A corta a 1 y 2 en C y D, respectivamente, y la paralela a CD por B corta 1 y 2 en E y F , respectivamente. Demuestre que CDB EAF . 41. (*) La Recta de Simson-Wallace. Sean X, Y y Z los pies de las alturas trazadas desde un punto P en el circunc rculo del ABC hacia AB, BC y CA, respectivamente. Demuestre que X, Y y Z estn alineados. a 42. (*) Sea P un punto exterior al cuadrado ABCD tal que AP C = 90 , Q es la interseccin o de AB y P C, y R el pie de la perpendicular por Q a CA. Demuestre que P , R y D estn a alineados. 43. En la gura 61, ABCD es un trapecio rectngulo tal que la circunferencia de dimetro a a AB (y centro O) es tangente a CD. Demostrar que O pertenece a la circunferencia de dimetro CD y que esta circunferencia es tangente a BA. a

Figura 61 44. El ABC es rectngulo en C, la circunferencia de centro O es tangente a cada uno de los a lados del ABC en los puntos P , Q y R (como se muestra en la gura 62), y se cumple que AP = 20 y BP = 6. Calcule OP . 43

Figura 62 45. En la gura 63 se muestran tres semicircunferencias, una de dimetro AB (de centro O a y radio r), otra de dimetro AO y la ultima de dimetro OB. Determine la razn entre a a o el radio de la circunferencia tangente a estas tres semicircunferencias y r.

Figura 63 46. El segmento AB es dimetro de un semic a rculo con centro en O. Un c rculo con centro en P es tangente a AB en O y tambin al semic e rculo. Otro c rculo con centro en Q es tangente a AB, al semic rculo y al c rculo de centro en P . Si AB = 2, cul es el radio a del c rculo con centro en Q?

Figura 64 Problemas de Refuerzo. 47. Los vrtices A y B de un tringulo equiltero ABC estn sobre una circunferencia de e a a a radio 1 y el vrtice C est en el interior de la circunferencia. Un punto D (distinto de B) e a que esta en la circunferencia es tal que AD = AB. La recta DC corta por segunda vez a la circunferencia en E. Encuentre la longitud del segmento CE. Ver gura 65. 48. (*) (OIM 2002, P-4) En un tringulo escaleno ABC se traza la bisectriz interior BD, con a D sobre AC. Sean E y F puntos sobre la recta BD tales que (AE CF ) BD, y sea M el punto sobre el lado BC tal que DM BC. Demuestre que EM D = DM F . 44

Figura 65 49. (*) (OMCC 2003, P-2) Sea S una circunferencia y AB un dimetro de ella. Sea t la recta a tangente a S en B y considere dos puntos C y D en t tales que B este entre C y D. Sean E y F las intersecciones de S con AC y AD y sean G y H las intersecciones de S con CF y DE. Demuestre que AH = AG. 50. (*) (The 59th Romanian Mathematical Olympiad District Round) Considere un cuadrado ABCD y un punto E sobre el lado AB. La diagonal AC corta al segmento DE en el punto P . La perpendicular por P a DE corta al lado BC en F . Probar que EF = AE + CF . 51. (*) Teorema de Arqu medes: En la gura 66, la regin delimitada por tres semio circunferencias mutuamente tangentes, es conocida como cuchilla de zapatero o rbelos. a Demostrar que las circunferencias sombreadas son congruentes.

Figura 66: Teorema de Arqu medes.

45

6.

Teorema de Thales y su rec proco. Semejanza de Tringua los.

Introduccin. o
Denicin o 1. Razn: se llama razn, al cociente de dos cantidades, expresadas en la misma magnitud, o o a por ejemplo b .
c 2. Proporcin: se llama proporcin a la igualdad de dos razones. Por ejemplo a = d , 16 a los o o b trminos a y d se les llama extremos y los trminos b y c se les llama medios, al trmino e e e d se le llama cuarta proporcional entre a, b y c en este orden.

Propiedades de las proporciones: a c 1. = si y slo si a c = b d. o b d 2. 3. 4. a c b d a b = si y slo si = o = . o b d a c c d a c ab cd = si y slo si o = . b d b d a c a+b c+d = si y slo si o = . b d ab cd

Paralelismo y proporcionalidad.
Denicin o 1. Un punto P AB divide al segmento AB en una razn dada r, si o
PA PB

= r.

Figura 67 2. Sean AB y CD dos segmentos, y sean P AB y Q CD, decimos que P y Q dividen a QA PA AB y CD en segementos proporcionales si P B = QB .

Figura 68
En algunos textos de geometr se utiliza la notacin de proporcin as a : b :: c : d que se lee a es a b como a o o c es a d.
16

46

Teorema de Thales. Si tres paralelas cortan a dos secantes entonces los segmentos que determinan en ellas son proporcionales. 17 Antes de demostrar el Teorema de Thales, se enunciarn dos teoremas que a pesar de su apaa rente sencillez es de mucha utilidad en problemas que involucran Areas y Proporcionalidad. Lema 1. Sea AB CD. Demuestre que: (ABC) = (ABD). ABC. Pruebe que:

Lema 2. Sea P un punto sobre el lado AB (o su prolongacin) del o AP (AP C) = PB (P BC)

. A continuacin se enuncian los pasos a seguir en la demostracin del teorema de Thales. o o Demostracin. Sean AA , BB y CC rectas paralelas que cortan a dos secantes en los puntos o A, A , B, B , C, C respectivamente (ver gura 69).

Figura 69: Teorema de Thales Pruebe que: 1. 2. AB (ABB ) = BC (BCB ) AB (A B B) = . BC (B C B)

3. (ABB ) = (A B B) y (BCB ) = (B C B). Con ayuda de las igualdades demostradas concluya que: AB AB = . BC BC Observacin Importante: Utilice las propiedades de las proporciones para demostrar las o equivalencias siguientes (interprtelas geomtricamente): e e AB AC AC AC AC AB = = = BC BC AB AB BC BC
El teorema de Thales puede enunciarse de manera general como sigue: Si tres o ms paralelas cortan a dos a o ms secantes entonces los segmentos que determinan en ellas son proporcionales. a
17

47

Corolario (Teorema de Thales en el tringulo). Toda recta paralela a un lado de un tringulo a a y que corte a los otros dos lados, divide a estos lados en segmentos proporcionales. Rec proco del Teorema de Thales. Si tres rectas cortan a dos secantes en segmentos proporcionales y dos de estas rectas son paralelas entonces las tres rectas son paralelas. Demostracin. Sean AA , BB y CC rectas que cortan a dos secantes en los puntos A, A , B, o AB AB B , C, C respectivamente, tales que AA CC y = . Por el punto B tracemos una BC BC recta paralela a AA , la cual interseca a A C en el punto D (ver gura 70). Entonces, por el AD AB AD AB = . De donde, = , as por las propiedades Teorema de Thales se tiene que: BC DC BC DC AC AC = , por lo que B C = B D + DC = DC y por tanto B D = 0, de las proporciones BC DC o equivalentemente B = D y por lo tanto, BB AA .

Figura 70: Rec proco del Teorema de Thales Corolario (Rec proco del Teorema de Thales en el tringulo.) Si una recta intercepta a dos lados de un tringulo en segmentos proporcionales entonces la recta es paralela al tercer a lado del tringulo. a Tringulos semejantes. Decimos que el a cual denotamos as ABC A B C , si: ABC es semejante al A B C (Ver gura 71), lo

AB AC BC = = AB AC BC y BAC = B A C , ABC = A B C , ACB = A C B . En los tres teoremas que se muestran a continuacin (los cuales son una consecuencia directa del o Teorema de Thales) se establecen las condiciones m nimas para demostrar que dos tringulos a son semejantes, a los cuales denominaremos: Criterios de Semejanza de Tringulos. a Primer criterio de semejanza de tringulos: Angulo-Angulo A-A. Si dos angulos de a un tringulo son congruentes con dos ngulos de otro tringulo, entonces los dos tringulos son a a a a semejantes. 48

Figura 71: Denicin de Semajanza de Tringulos. o a Demostracin. Supongamos que en el ABC y A B C se tiene que ABC = A B C o y ACB = A C B , entonces BAC = B A C (Por la suma de angulos internos en un tringulo). a Sea D AB y E AC tales que AD = A B y AE = A C , dado que DAE = BAC = B A C , se sigue por L-A-L que ADE = A B C , por consiguiente ADE = A B C = ABC, de donde DE BC (por ser iguales los angulos correspondientes) y por el teorema de Thales AC AB = AD AE y por consiguiente AC AB = (1) AB AC Sea F BC tal que DF AC, entonces F C = DE = B C (porque DECF es paralelogramo y por ser ADE = A B C ) y por el teorema de Thales BA BC = DA FC o lo que es lo mismo AB BC = AB BC Luego, de (1) y (2) se tiene que: AB AC BC = = . AB AC BC As se ha demostrado que los tres pares de angulos son congruentes y los tres pares de lados , son proporcionales, por lo tanto, ABC A B C . Segundo criterio de semejanza de tringulos: L-A-L. Si un angulo de un tringulo es a a congruente con otro angulo de otro tringulo y los lados que comprenden al ngulo en el primer a a tringulo son respectivamente proporcionales a los lados que comprende al angulo en el segundo a tringulo, entonces los dos tringulos son semejantes. a a AB AC Demostracin. Suponga que el BAC = B A C y que o = . Considere los punAB AC tos D y E, como en la demostracin del teorema anterior. Entonces por el criterio L-A-L, o (2)

49

ADE = A B C , de lo cual se deduce que ADE = A B C . Por otra parte teneAB AC mos que: = , y al aplicar el rec proco del teorema de Thales, se puede armar que AD AE DE BC, de lo cual a su vez se deduce que ADE = ABC, por angulos correspondientes entre paralelas. Finalmente por transitividad se concluye que ABC = A B C . Por lo tanto, ABC A B C (Por el criterio A-A.) Tercer criterio de semejanza de tringulos: L-L-L. Si los tres lados de un tringulo son a a respectivamente proporcionales a los tres lados de otro tringulo, entonces los dos tringulos a a son semejantes. AC BC AB = = y como antes sean D y E Demostracin. Por hiptesis se tiene que: o o AB AC BC puntos sobre AB y AC respectivamente tales que AD = A B y AE = A C . Entonces por el rec proco del teorema de Thales se tiene que DE BC y por consiguiente el ABC = ADE y el ACB = AED, de donde ABC ADE (por el criterio A-A). Por ende BC BC BC AB = , luego por transitividad = , de donde DE = B C . En consecuencia AD DE DE BC ADE = A B C (por el criterio L-L-L), de lo cual se sigue que A B C = ADE y A C B = AED, y por transitividad A B C = ABC y A C B = ACB =. Por lo tanto, A B C ABC (Por el criterio A-A.)

Ejercicios
1. Sean AB y CD las bases del trapecio ABCD, cuyas diagonales son perpendiculares. Si se sabe que AD = 13, AE = 12 y CE = 4 encuentre las longitudes de CD y AB. 2. En la gura 72, el ABC es equiltero, sus lados tienen longitud 3 y P A es paralela a a BC. Si P Q = QR = RS, encontrar la longitud de CS.

Figura 72 3. Sea ABCD un trapecio de bases BC y AD, sus diagonales se cortan en E. Si BE = 3, ED = 4 y CE = 2, determine la medida de AE. 4. Las bases de un trapecio miden 3 y 5, y si su altura mide 4. Encontrar la distancia desde el punto de corte de las diagonales hasta la base mayor.

50

5. En la gura adjunta, el ABC es rectngulo en A y el ADB a es rectngulo en D. El punto E es el punto de interseccin de a o los segmentos AD y BC. Si AC = 15, AD = 16 y BD = 12, calcule el area del ABE.

6. El ABC es rectngulo en B. Se dibuja un rectngulo BEDF con D sobre la hipotenusa, a a 1 BC E y F sobre BC y AB, respectivamente. Si AB = 1, demuestre que BE = 1DE . 7. Considrese los puntos A, B, C y D tales que A y B estn sobre el segmento OC y e a OD respectivamente, donde O es el centro de la circunferencia de radio r (Ver gura 73). Si OA OC = r2 = OB OD, demuestre que el AOB DOC y que CD = r2 AB.18 OA OB

Figura 73 8. Sobre la circunferencia de centro O, se trazan los dimetros AB y CD tales que AB CD. a Sea P un punto sobre el arco CBD y Q el punto de interseccin de las cuerdas AP y o CD. Si DO = 1, demuestre que AP AQ = 2. 9. Un segmento de recta AB es divido por los puntos interiores K y L de manera que AL2 = AK AB. Sea P un punto exterior al segmento AB tal que AP = AL. Pruebe que KP L = LP B. Figura 74.

Figura 74
18

La medida del segmento CD se denomina Distancia Inversa.

51

10. En la gura 75, AB y AC son tangentes a la circunferencia, y CE BD, siendo BD un dimetro. Probar que BE BO = AB CE. a

Figura 75 11. Demostrar que 1 1 1 + = si se cumple que AX AX BY AZ BY CZ. (Ver gura 76.)

Figura 76 12. En la gura 77, el ABC es rectngulo. Se construyen exteriormente los cuadrados a ABEF y BCP Q. Demostrar que BM = BN .

Figura 77: .

52

13. Sean O, P y R los centros de las tres circunferencias. Si OR = r y Q es la interseccin o 2 de P O con la circunferencia de centro R, demuestre que OP OQ = r . Ver gura 78.

Figura 78 14. Si en un tringulo rectngulo se traza la altura correspondiente a la hipotenusa, entonces: a a a) Los dos nuevos tringulos que resultan, son semejantes entre si y semejantes al a tringulo original. a b) La altura es media proporcional hipotenusa.
19

entre los segmentos que ella determina sobre la

c) Cada cateto es media proporcional entre la hipotenusa y la proyeccin del cateto o sobre la hipotenusa. d ) Demuestre el teorema de Pitgoras. a 15. Si dos tringulos tienen sus lados respectivamente paralelos o respectivamente perpendia culares, entonces los dos tringulos son semejantes. a 16. Las alturas, las bisectrices y las medianas homlogas de dos tringulos semejantes estn o a a en la misma razn que sus lados homlogos. o o
BC CA AB 17. Sean ABC y A B C dos tringulos semejantes con A B = B C = C A = k. Demuestre a que: la razn entre los per o metros de los tringulos es k y que la razn entre sus areas es a o 2 k .

18. Teorema de Menelao. Dado el ABC, sea P un punto sobre la recta AB, Q un punto sobre la recta BC, R un punto sobre la recta CA. Si los puntos P , Q, R estn alineados a AP BQ CR entonces = 1. P B QC RA Para demostrar este teorema, sea W un punto sobre la recta P QR tal que BW AC: a) Demuestre que los tringulos AP R y BP W son semejantes. a
b Si b es una magnitud tal que a = c , entonces decimos que b es media proporcional entre a y c,o de manera b equivalente: b es media proporcional entre a y c si y solo si b2 = a c. 19

53

Figura 79: Teorema de Menelao. b) Demuestre que los tringulos CQR y BQW son semejantes. a AP BQ CR c) De los literales a) y b) deduzca que = 1. P B QC RA

19. Teorema de Ceva. Dado el ABC, sea P un punto sobre el recta AB, Q un punto sobre la recta BC y R un punto sobre la recta CA. Si las rectas AQ, CP , BR concurren, entonces AP BQ CR = 1. P B QC RA

Para demostrar este teorema, sean W y V los puntos de interseccin de la recta que pasa o por B paralela a AC, con las rectas CP y AQ, respectivamente. a) Demuestre que b) Demuestre que AP C BW P BP W y que RCP y que AQC V QB.

BV P RAP . AP BQ CR = 1. c) Utilice los literales a) y b) para probar que P B QC RA

20. Si dos cuerdas se interceptan en el interior de una circunferencia entonces el producto de las medidas de los segmentos determinados por el punto de interseccin en una de las o cuerdas es igual al producto de las medidas de los segmentos determinados en la otra cuerda. 21. Si dos segmentos se interceptan en un punto que esta en el interior de los dos segmentos y el producto de las medidas de los segmentos determinados por el punto de interseccin o en el primer segmento es igual al producto de las medidas de los segmentos determinados por el punto en el segundo segmento,entonces los extremos de los segmentos estn sobre a una circunferencia. 22. Si desde un punto P exterior a una circunferencia se trazan dos semirrectas secantes que cortan a la circunferencia en los puntos A, B y C, D respectivamente, entonces P A P B = P C P D.

54

23. Si desde un punto P se trazan dos semirrectas con los puntos A, B sobre una y los puntos C, D sobre la otra, tales que P A P B = P C P D, entonces los puntos A, B, C, D estn a sobre una circunferencia. 24. Si desde un punto exterior a una circunferencia se trazan dos semirrectas, una tangente y la otra secante, entonces el segmento entre el punto y el punto de tangencia es media proporcional entre los segmentos determinados entre el punto exterior y los puntos de interseccin de la secante con la circunferencia. 20 o 25. Si P es un punto sobre el mismo plano que una circunferencia de centro O y radio r, y d es la distancia del punto P al centro O de la circunferencia, demuestre que: a) Si P est en el interior de la circunferencia, entonces la potencia de P es r2 d2 . a b) Si P est en el exterior de la circunferencia, entonces la potencia de P es d2 r2 . a c) Si P est sobre de la circunferencia, entonces la potencia de P es cero. a 26. (*) (IV OMCC, P-4) Sea ABC un tringulo, D el punto medio de BC, E un punto sobre a el segmento AC tal que BE = 2AD y F el punto de interseccin de AD con BE. Si o CAD = 60, encuentre la medida de los angulos del F EA. 27. (*) Sea ABCD es un trapecio con AD BC. M y N son los puntos medios de CD y BC, respectivamente, y P el punto comn de las rectas AM y DN . Si P M = 1 , demuestre que u AP 4 ABCD es paralelogramo. 28. Dado el ABC se construye un cuadrado P QRS con P en AB, Q en AC, R y S en BC. Sea H el pie de la altura desde A hacia BC. Demuestre que: a) 1 1 1 = + PQ AH BC b) (ABC) = 2(P QRS) si y slo si AH = BC. o

29. Sea P un punto en el interior del ABC. Se trazan por P las paralelas a los lados del tringulo, que queda dividido en tres tringulos y tres paralelogramos. Si las reas de los a a a tres tringulos de la subdivisin son, en algn orden, 9, 16 y 25, hallar el rea del ABC. a o u a Problemas de Refuerzo.

30. (*) En la gura anexa, BC = CD = DE = EA = x y AEB = 90. Demuestre que ABC + ACD + ADE = 90.

Los problemas anteriores nos permite establecer la siguiente denicin de Potencia de un punto con o respecto a una circunferencia: La potencia de un punto P con respecto a una circunferencia de centro O y radio r es el producto P A P B, donde A y B son los puntos de interseccin de la circunferencia con una recta o que pasa por P .

20

55

31. Las tres circunferencias de la gura 80 tienen el mismo radio r, sus centros son colineales y la circunferencia de centro O2 es tangente a las otras dos. Por A se traza una tangente a la circunferencia de centro O3 . Obtenga el valor del segmento BC en funcin de r. o

Figura 80 32. Sea ABCD un rombo, con AC = 6 y BD = 8. Se construyen exteriormente los cuadrados ADEF y CDHG, cuyos centros son O1 y O2 , respectivamente (Vea gura 81). Calcular la medida del segmento O1 O2 .

Figura 81 33. Dado un paralelogramo ABCD, se trazan dos circunferencias tangentes externamente en P , y tales que la primera es tangente internamente al ABC y la otra es tangente internamente al CDA, como en la gura 82. Demuestre que B, P y D estn alineados. a 34. Sea ABCD un cuadrado con P y Q sobre AB y BC tales que BP = BQ. Sea H el pie de la perpendicular de B a P C. Demuestre que DHQ = 90. 56

Figura 82 35. En un ABC el CAB = 120. Encuentre la medida de la bisectriz interna del CAB en funcin de los lados adyacentes. o 36. El ABC tiene lados de 13, 14 y 15 unidades. El A B C est dentro del ABC con a lados paralelos a los de ste y a 2 unidades de distancia de los lados del mismo. Calcule e (ABC) (A B C ). 37. (*) (Asitico Pac a ca) Sea ABC un tringulo y D el pie de la altura con respecto a A. a Sean E y F puntos en una recta que pasa por D (distintos de D) tales que AE CE y AF BF . Sean M y N los puntos medios de BC y EF , respectivamente. Demuestre que AN N M .

57

7.

Puntos y Rectas Notables del Tringulo. a

MEDIANAS
Denicin: En un tringulo, una mediana es el segmento de recta que une un vrtice con el o a e punto medio del lado opuesto. Teorema: Las tres medianas de un tringulo concurren en un punto llamado el Centroide 21 a del tringulo y usualmente es denotado por G. Adems, las medianas de cortan mutuamente a a en razn 2:1. o Demostracin: Dado el ABC sean A , B , C , los puntos medios de BC, CA, AB, respeco tivamente. Dena G como la interseccin de BB con CC . Por el teorema de la base media, o BC BC y 2B C = BC; observe que BCG B C G, con razn de semejanza 2, por lo o que GC GB = =2 GB GC Anlogamente, si G = AA BB se cumple a G A G B = =2 G B GA As G y G dividen al segmento BB en dos segmentos cuya razn es 2:1, por lo que G = G , , o lo cual implica que AA , BB , CC concurren y GB GC GA = = =2 GA GB GC

MEDIATRICES
Denicin: Dado un segmento AB, la mediatriz del segmento es el lugar geomtrico de puntos o e que equidistan de A y B, i.e., un punto P est sobre la mediatriz de AB si y slo si P A = P B. a o Teorema: La mediatriz de AB es una recta l perpendicular a AB y que pasa por su punto medio. Demostracin: Sea M el punto medio de AB, y l pasa por M y l AB. En primer lugar o se probar que todos los puntos de l satisfacen la denicin de mediatriz: Por denicin de a o o punto medio M A = M B. por lo que claramente M pertenece a la mediatriz de AB; sea P un punto de l distinto de M , por criterio LAL, P M A P M B por lo que P A = P B. Ahora, cabe preguntarse si existe algn punto fuera de l que tambin cumpla la denicin: suponga u e o P tal que P A = P B, esto implica que P AB es issceles, y entonces P AB = P BA; o si M es la proyeccin de P sobre AB, por criterio ALA P AM P BM , lo cual implio ca que M A = M B, es decir que M = M , y esto obliga a que P est sobre l (ya que P M = l). e Teorema: Las mediatrices de un ABC concurren en un punto que equidista de los vrtices e del tringulo, llamado el Circuncentro del ABC a
21

Tambin conocido como Geocentro, Centro de Gravedad, Baricentro, o ms formalmente Equibaricentro. e a

58

Figura 83: Concurrencia de Mediatrices, Circuncentro y Circunc rculo. Usualmente, el circuncentro es denotado por O, y R representa la distancia del circuncentro a los vrtices e R = OA = OB = OC A esta distancia se le llama Circunradio del ABC. As O es el centro de una circunferencia , que pasa por A, B, C, cuyo radio es R, llamada el Circuncrculo del ABC.22 Demostracin: Sea O la interseccin de las mediatrices de AB y BC, por el teorema anterior, o o como O pertenece a la mediatriz de AB se cumple OA = OB, y como tambin pertenece a la e mediatriz de BC, OB = OC; entonces OC = OA, y utilizando de nuevo el teorema anterior, O debe pertenecer a la mediatriz de CA. As las tres mediatrices concurren en O, y este punto , equidista de los vrtices del ABC. e Corolario: Dado un tringulo, existe una circunferencia que pasa por los tres vrtices (el cira e cunc rculo); adems, esta circunferencia es unica. a Una observacin importante es que la mediatriz del lado de un tringulo NO siempre pasa por o a el vrtice opuesto; de hecho, esto slo se da si el tringulo es issceles. e o a o

ALTURAS
La altura es un concepto que est intr a nsecamente relacionado con la distancia de un punto a una recta; la altura es la recta que debe trazarse para determinar esta distancia, i.e., es una recta que pasa por el punto y es perpendicular a la recta. A la interseccin entre la altura y la o recta generalmente se le llama pie de la altura, o tambin (ms formal) proyeccin del punto e a o sobre la recta. En particular, para tringulos, deniremos la altura de la siguiente forma: a Denicin: Dado un tringulo, una altura es una recta que pasa por un vrtice y es perpeno a e dicular al lado opuesto.
22

En ocasiones, denotaremos a esta circunferencia por (ABC).

59

Es importante observar que el pie de la altura NO siempre pertenece a un lado; de hecho, una altura puede estar al interior de un tringulo, coincidir con un lado, o estar completamente a afuera de un tringulo. a Teorema: Las alturas de un tringulo concurren en un punto, llamado el Ortocentro del tringua a lo, usualmente denotado por H.23 Demostracin: Dado el ABC, se construyen los puntos A1 , B1 , C1 , tales que ABA1 C, o BCB1 A, CAC1 B son paralelogramos. Observe que el ABC es el tringulo medial del A1 B1 C1 , a y que las alturas del ABC son las mediatrices del A1 B1 C1 ; como las mediatrices de un tringulo concurren (en este caso, las del A1 B1 C1 ), las alturas del ABC concurren. a La altura tambin puede escribirse en trminos de lugar geomtrico: e e e Teorema: La recta l es perpendicular a AB si y slo si AL2 LB 2 es constante. Es decir, que o una recta perpendicular a AB es el lugar geomtrico de los puntos L que satisfacen la condicin e o anterior. Demostracin: sea P la interseccin de l con AB, y L un punto arbitrario sobre l; por Pitgoo o a 2 2 2 2 ras se tiene AL LB = AP P B , y el trmino derecho de la igualdad es constante. La e otra direccin de la implicacin se prueba por contradiccin. o o o De esa denicin tambin puede fabricarsele una demostracin del teorema anterior, sin emo e o bargo, no se aborda porque la prueba se basa en un resultado sosticado llamadado Teorema de Steiner.24

BISECTRICES
Denicin: La bisectriz de un ngulo es una recta que divide al angulo en dos angulos de o a igual magnitud. Teorema: El lugar geomtrico de puntos que equidistan de dos rectas dadas, generan un par e de rectas perpendiculares llamadas bisectriz interna y bisectriz externa del angulo formado por las rectas. Demostracin: Suponga que las rectas se cortan en un punto O; sean a, b las rectas dadas, y o P un punto que equidista de ellas; si A y B son las proyecciones de P sobre a y b, respectivamente, entonces P A = P B. Observe que por criterio LLL (utilizando Pitgoras previamente), a OAP OBP , por lo que P OA = P OB, i.e., P pertenece a la bisectriz del AOB. Claramente aqu se dan dos casos, recuerde que para denir el angulo entre a y b se utilizan uni camente semi-rectas, por lo que las rectas a y b denen cuatro angulos, que por parejas pueden
El tringulo formado por los pies de las alturas de un ABC es llamado el tringulo rtico del ABC. a a o Sean l, m, n, tres rectas perpendiculares a los lados del AB, BC, CA del ABC, respectivamente. Sean L, M , N , puntos arbitrarios sobre l, m, n, respectivamente. Entonces las rectas l, m, n concurren si y slo si o AL2 + BM 2 + CN 2 = N A2 + LB 2 + M C 2 .
24 23

60

ser opuestos por el vrtice o suplementarios; de estos se escoge cualquiera de ellos como referene cia, entonces, si AOB coincide con ste o con el opuesto por el vrtice, la recta P O es llamada e e bisectriz interna, y en caso contrario, bisectriz externa. As el lugar geomtrico son dos rectas, , e y su perpendicularidad se basa en los pares de angulos que son suplementarios. Finalmente, si a b, el lugar geomtrico es una recta paralela a a y b que se ubica entre ellas a igual distancia de e ambas (este es un caso extrao de bisectriz interna, sin embargo, en ocasiones es util tener esta n convencin en mente; peor an, la bisectriz externa es una recta ideal llamada recta al innito). o u Teorema: Las bisectrices internas de un ABC concurren en un punto, llamado el Incentro del ABC, usualmente denotado por I. La distancia de I a los tres lados del tringulo es igual a a un nmero r, llamado el Inradio del ABC, y de aqu que la circunferencia de centro I y u radio r sea tangente a los lados del tringulo; dicha circunferencia es llamada el Incrculo del a ABC.25

Figura 84: Concurrencia de Bisectrices Internas, Incentro e Inc rculo. Demostracin Sea I la interseccin de las bisectrices internas de A y B (obviamente, I o o est en el interior del ABC); como I pertenece a la bisectriz interna del A, por el teorema a anterior dist(I, AB) = dist(I, AC), y anlogamente, como I pertenece a la bisectriz interna del a B, dist(I, AB) = dist(I, CB); entonces dist(I, AC) = dist(I, CB), y de nuevo por el teorema anterior y dado que I est al interior del tringulo, I pertenece a la bisectriz interna del C. a a As las tres bisectrices internas concurren en un punto que equidista de los lados del tringulo. , a Es importante notar que las interseccin de una bisectrices interna con el lado opueto del o tringulo NO siempre coincide con el puntos de tangencia del inc a rculo;26 de hecho, esto ocurre slamente si el tringulo es issceles. o a o Corolario: Dado un tringulo, existe una circunferencia que es tangente interiormente a los a tres lados (el inc rculo); adems, esta circunferencia es unica.27 a
En algunas ocasiones denotaremos al inc rculo por (ABC). En la gura, el ABC es llamado tringulo tangencial del DEF . a 27 Existen 3 circunferencias ms que son tangentes a los tres lados del tringulo, llamados exc a a rculos; estas circunferencias se ubican en el exterior del tringulo. a
26 25

61

Ejercicios
1. Las reas de los seis tringulos AGB , AGC , BGA , BGC , CGA , CGB son iguales e a a 1 iguales a un 6 del rea del tringulo ABC. a a

Figura 85 2. Los cuatro tringulos AB C , BC A , CA B , A B C ,28 son congruentes entre si y semea jantes al ABC con razn de semejanza 1 . o 2 3. El centroide del ABC coincide con el centroide del tringulo medial A B C . Adems, a a estos dos tringulos tienen lados correspondientes paralelos (tringulos homotticos). a a e 4. En la gura 86, G es el centroide. Si GD = 2 y el area sombreada vale 5, calcule AD y el (ABC).

Figura 86 5. Demostrar que las paralelas a los lados de un cada lado en tres partes iguales. ABC, trazadas por el centroide G dividen

6. ABCD es un paralelogramo de centroide (baricentro) E, M es el punto medio de AD, y F es la interseccin de AC con BM . Si el rea de ABCD es 1, calcule el rea del o a a cuadriltero DEF M . a 7. En el ABC, se traza la mediana AM . Demostrar que si BM = AM , entonces el tringulo es rectngulo en A. a a
28

El

A B C es llamado el tringulo medial del a

ABC.

62

8. La suma de las distancias del centroide a los puntos medios de los lados de un tringulo a es 20. Calcule la suma de las medianas del tringulo. a 9. La mediana tiene longitud menor que la semisuma de los lados adyacentes, es decir AA < b + c, BB < c + a, CC < a + b. 10. Dado el ABC, sean D y E puntos variables sobre los lados AB y AC respectivamente tales que BC DE. Entonces, la mediana AA puede denirse como el lugar geomtrico e 29 de los puntos P tales que P CD BE. 11. Siempre es posible construir un tringulo XY Z con las medianas AA , BB , CC de un a ABC dado. Adems, los segmentos que unen el centroide del XY Z con sus vrtices a e son iguales a la mitad de los lados del ABC. 12. En el ABC, AB = BC y la mediatriz de BC interseca a la mediana BM en L. Si LCB = 25, determine la medida del LAC. 13. Ley del Seno. Dado un ABC, se cumple que senB senC 1 senA = = = a b c 2R 14. Las reexiones de H con respecto a los lados del ABC caen sobre el circunc rculo del mismo, es decir HHa = Ha X y anlogo para los otros lados. a

Figura 87 15. Las reexiones de H con respecto a los puntos medios de los lados del tringulo, caen a sobre el circunc rculo del mismo.
29

Si D = A se dene P = A, y cuando D = B entonces P es punto medio de BC.

63

16. Si O y H son el circuncentro y el ortocentro de un BAH = CAO. 17. La altura AHa es bisectriz del Hb Ha Hc . 18. Los circunc rculos de ABC, ABH, BCH,

ABC, respectivamente, entonces

CAH tienen igual radio.

19. La perpendicular trazada desde A al lado Hb Hc del tringulo rtico, pasa por el circuna o centro del ABC. 20. A, B, C y H forman un cuadriltero ortocntrico, es decir que cada punto es el ortocentro a e del tringulo formado por los otros tres. a 21. El ortocentro de un tringulo est al interior, sobre un vrtice, o afuera del tringulo, si a a e a el tringulo es acutngulo, rectngulo, u obtusngulo, respectivamente. a a a a 22. El circuncentro del ABC es el ortocentro del tringulo medial a ABC .

23. Sea O el circuncentro del ABC. Si AOC = 100 y OCB = 30, determine la medida de los angulos del ABC. 24. Hallar los ngulos de un tringulo cuyo tringulo ortico tiene angulos de 20, 50 y 110. a a a 25. Sea ABC un tringulo obtusngulo de circuncentro O y altura AD. Si OAB = 25 y a a OCB = 15, calcule el DAB. 26. El ABC de circuncentro O y altura BD. Si DAB = 35 y OBD = 10 encontrar los angulos del tringulo ABC. a 27. En la gura 88, AB es dimetro de la circunferencia. Si X es la interseccin de CG con a o AB, calcular el CXB.

Figura 88

64

28. En el ABC, se trazan la altura AH y la mediana BM . Demuestre que el issceles. o 29. Un

M HC es AB . 4

ABC es rectngulo en C, A = 75 y CH es altura. Demuestre que CH = a

30. Sea O el circuncentro del ABC con C = 45 y sea D el pie de la altura desde A. Calcule la medida del ODC. 31. Dado el ABC issceles con A = 90, sean P y Q son puntos dentro del tringulo tales o a que BP = AQ y AP = CQ. Si BP y CQ se cortan en R, demostrar que AR P Q. 32. Se ubican los puntos M y K sobre los lados BC y CD del cuadrado ABCD, respectivamente, de modo que M C = KD. Sea P la interseccin de M D y BK, demuestre que o AP M K. 33. Sean D, E, F los puntos de tangencia del inc rculo sobre los lados BC, CA, AB del ABC. Demuestre que se cumplen las siguientes relaciones, donde s denota el semiper metro del tringulo: a AE = AF = s a BD = BF = s b CD = CE = s c 34. El ortocentro del ABC es el incentro de su tringulo ortico. a

35. Dado un ABC, su tringulo ortico y su tringulo tangencial tienen lados correspona a dientes paralelos (tringulos homotticos). a e 36. Las bisectrices exteriores de B y C, junto con la bisectriz interior de A, concurren en un punto, llamado el Excentro con respecto al vrtice A, usualmente denotado por Ia . e Este punto es equidistante a los lados del ABC, dicha distancia es el Exradio respecto a A, usualmente denotado por ra . As la circunferencia de centro Ia y radio ra es tangente , exteriormente a los lados del ABC, y es llamada el Excrculo respecto a A.30 37. I es ortocentro del Ia Ib Ic . Adems se cumple: a AX = AZ = s BX = BY = s b CY = CZ = s c 38. En un ABC, la bisectriz exterior del ABC y la bisectriz exterior del BCA se cortan en D. La paralela a BC por D corta a AC en L y a AB en M . Si LC = 5 y M B = 7, hallar LM . 39. El ABC es rectngulo en A. Si I es el incentro, calcular BIC. a

40. En un ABC, el ABC CAB = 90. Sean D y E los pies de las bisectrices interior y exterior del BCA respectivamente. Demuestre que CD = CE.
30

Anlogamente se denene los exc a rculos con respecto a los otros vrtices. e

65

Figura 89 41. En el ABC, AB < AC, AD es bisectriz, y E es un punto en AB tal que el EDB = 90. El punto F sobre AC es tal que el BED = DEF . Demuestre que el BAD = F DC. 42. En el ABC se trazan las bisectrices interiores BD y CE tales que D es el punto sobre AC, E es el punto sobre AB, 2BDE = 3B y CED = 2B. Calcular los ngulos del a ABC. 43. Dado el ABC con A = 90, sea D el pie de la perpendicular desde A. Sean adems I a y J los incentros respectivos de ABD y ACD. Demostrar que la bisectriz del BAC es perpendicular a IJ. 44. Un tringulo es issceles si cumple alguna de las siguientes condiciones: a o a) Dos medianas son iguales. b) Dos alturas son iguales. c) Dos bisectrices son iguales.31 45. Teorema de la Bisectriz: Dado el ABC, sean P y P sobre BC. Se cumple que AP y AP son la bisectriz interna y la bisectriz externa del A si y slo si o BA BP BP = = PC AC PC Sugerencia: Para demostrar la primera igualdad, trace CD gacin de AB. o
31

AP con E sobre la prolon-

Este caso es aparentemente tan sencillo como los anteriores, pero realmente es un resultado muy complicado y recibe el nombre de Teorema de Steiner-Lehmus.

66

46. (*) De acuerdo con los datos de la grca 90, calcular el valor de AB. a

Figura 90 47. Dos circunferencias son tangentes internamente en P , y una cuerda AB de la circunferencia de radio mayor es tangente en Q a la otra circunferencia. Ver gura 91. a) Demuestre que P Q es bisectriz del AP B. b) Llame A y B a las otras intersecciones de P A y P B con la circunferencia de radio menor y suponga que AB = 15, P A = 3 y P B = 2; calcule AQ y BQ.

Figura 91 48. (*) Sea ABC un tringulo tal que las medianas respectivas a B y C son perpendiculares. a Demuestre que se cumple la relacin. o 5BC 2 = CA2 + AB 2 . 49. (*) Teorema de Poncelet: Demuestre si 90, entonces 2(r + R) = b + c. ABC es un tringulo rectngulo con A = a a

67

Problemas de Refuerzo. 50. (*) Sea ABCD un paralelogramo. Q es el punto medio de AD, F el pie de la perpendicular por B sobre QC. Probar que AF = AB. 51. Dado el rombo ABCD, se trazan las bisectrices internas de DAC, CAB, BCA, ACD, y cortan a DC, CB, BA, AD en P , Q, R, S, respectivamente. Demuestre que P QRS es un rectngulo. a 52. (*) Sea ABCD un cuadriltero tal que AB = CD. Las mediatrices de AC y BD se cortan a en P . Probar que P AC = P CA = P BD = P DB. 53. (*) ABC es un tringulo y P un punto en su interior. Sean A , B y C las reexiones de a P sobre BC, CA y AB, respectivamente. D, E y F son los pies de las perpendiculares respectivos desde A, B y C hacia B C , C A y A B . Probar que AD, BE y CF son concurrentes. 54. (*) (Arnoldo Aguilar) En la gura 92, ABGH, BCF G y CDEF son cuadrados. Si I es el centro de ABGH y J = DH BG, demuestre que I, J y F estn alineados. a

Figura 92 55. (*) (Arnoldo Aguilar) Sea ABC un tringulo equiltero. M y N son los puntos medios de a a AB y BC, respectivamente. Exteriormente al ABC se construye un tringulo rectngulo a a issceles AP C, con AP C = 90 . Si I es la interseccin de AN y M P , demuestre que o o CI es la bisectriz de ACM .

56. (*) En la gura adjunta, el ABC es tal que A = 90 y B = 60. Cul es el radio de la circunferencia? a

57. (*) Dado el paralelogramo ABCD, sea M el punto medio de AB, y N la interseccin de o CD con la bisectriz interna del ABC. Demuestre que M C BN si y slo si AN es o bisectriz del DAB. 68

58. (*) En el ABC, se sabe que los vrtices B, C, el circuncentro O y el ortocentro H del e ABC estn todos sobre una misma circunferencia. a a) Calcule el valor de A. b) Demuestre que el incentro tambin pertenece al circunc e rculo de BCOH. 59. (*) Sea ABC un tringulo de ortocentro H. Sean P y Q los pies de las perpendiculares a desde H a las bisectrices interior y exterior de A, respectivamente. Si M es el punto medio de BC, mostrar que P , Q y M estn alineados. a 60. (*) En un tringulo ABC, sea M el punto medio de BC. Si se cumple que AB = AC y a adems M AC + ABC = 90 , hallar BAC. a 61. (*) Sea ABC un tringulo y U un punto de su circunc a rculo tal que AU es bisectriz. Las mediatrices en AB y AC cortan a AU en X y Y . Sea T la interseccin de BX con CY . o Demostrar que AU = T B + T C. 62. (*) (The 59th Romanian Mathematical Olympiad Final Round) Sea ABCD un rectngulo a de centro O con AB = BC. La perpendicular en O a BD corta a las l neas AB y BC en los puntos E y F , respectivamente. Sean M y N los puntos medios de los segmentos CD y DA, respectivamente. Probar que las l neas rectas F M EN . 63. (*) Sea ABC un tringulo rectngulo, con A = 90 . Sea D un punto en su interior tal a a que DAC = DCA = DBC = , y AC = BD. Determine el valor de . 64. (*) Sea ABC un tringulo y M un punto tal que M AB = 10, M BA = 20, M AC = a 40 y M CA = 30. Probar que el ABC es issceles. o 65. (*) En la gura 93, ABCD y P QRS son cuadrados, ABP BCQ CDR DAS y los los radios de las cinco circunferencias son iguales a r. Si a es el lado del cuadrado ABCD, determine r en funcin de a. o

Figura 93

69

66. (*) Recta de Euler. El centroide G, el ortocentro H y el circuncentro O de un tringulo a estn alineados, y adems GH = 2GO. a a 67. Circunferencia de los 9 puntos:32 Dado un ABC de ortocentro H, se cumple que los puntos medios de los lados, los pies de las alturas, y los puntos medios de HA, HB, HC, se ubican sobre una misma circunferencia. Adems, el centro de esta circunferencia a R es el punto medio de HO y su radio es 2 , donde O y R son el circuncentro y el circunradio del tringulo. a Para demostrar este resultado se sugiere seguir los siguientes pasos: a) Si Ha es el pie de la altura trazada desde A, demuestre que la reexin de H con o respecto a Ha pertenece a circunc rculo del ABC. Resultados similares se cumplen para Hb y Hc . b) Si A es el punto medio de BC, demuestre que la reexin de H con respecto a A o pertenece al circunc rculo del ABC. c) De los resultados anteriores, observe que hay 9 puntos sobre el circunc rculo del ABC: los vrtices, las reexiones de H con respecto a los pies de las alturas, y e las reexiones de H con respecto a los puntos medios de los lados; a partir de esto, concluya que los puntos medios de los segmentos que van de H a estos 9 puntos, tambin deben pertenecer en una misma circunferencia. e d) Concluya adems que el centro de esta nueva circunferencia es el punto medio de HO. a Otro camino de solucin es el siguiente: o a) Sea A B C el tringulo medial del ABC. Pruebe que A B C = BHa C y a concluya que Ha A B C es un cuadriltero c a clico; los mismo debe cumplirse para Hb y Hc . b) Sea X el punto medio de HA. Demuestre que B A C +B XC = 180 y concluya que XC B A es un cuadriltero c a clico; lo mismo debe cumplirse para los puntos medios de HB y HC. c) De lo anterior, concluya que los pies de las alturas y los puntos medios de los segmentos que van desde H hasta los vrtices del ABC, se ubican sobre el circunc e rculo del ABC . d) Si N es el circuncentro del A B C , demuestre que N , O, G forman la recta de Euler del A B C y utilice sus propiedades para probar que N es el punto medio de HO. 68. El area del ABC, denotada por [ABC], cumple: base altura 2 ab senC bc senA ca senB abc [ABC] = = = = 2 2 2 4R [ABC] = sr [ABC] = [ABC] =
32

s(s a)(s b)(s c). (Frmula de Hern). o o

Tambin conocida como Circunferencia de Feuerbach. e

70

69. El circunradio, el inradio y los exradios de un tringulo cumplen: a 4R = ra + rb + rc r [ABC] = ra (s a) = rb (s b) = rc (s c) r = ra = rb = rc = 70. Dado el (s a)(s b)(s c) s s(s b)(s c) sa s(s a)(s c) sb s(s a)(s b) (s c)

ABC, sea I el incentro e Ia el excentro respecto a A.

a) Demuestre que BICIa es un cuadriltero c a clico. b) Si M es la interseccin de IIa con el circunc o rculo del tringulo (M = A), demuestre a que dicho punto es el circuncentro de BICIa . c) Sea M el punto diametralmente opuesto a M en el circunc rculo, y sea P la proyeccin o de I sobre AB. Demuestre que M CM AIP . d) Sea O el circuncentro del tringulo Calcule la potencia de punto de I con respecto a al circunc rculo, y utilizando los resultados anteriores deduzca la Frmula de Euler : o OI 2 = R2 2Rr. e) A partir de la frmula de Euler demuestre que R 2r. o

71

8.

Solucin a Problemas Selectos. o

Teoremas Fundamentales del Tringulo. a


1. En la gura 94, ABDE es un cuadrado y BCD es un tringulo issceles con BD = DC. a o Si ABC = 160, determinar la medida de AEC.

Figura 94 Solucin: DBC = DCB = 160 ABD = 70, de donde se obtiene que BDC = 40 o y EDC = EDB + BDC = 130. Como EDC es issceles, entonces DEC = o DCE = 25. Por lo tanto AEC = 90 DEC = 65. 2. Hallar la suma de los ngulos + + + en la gura 95. a

Figura 95 Solucin: CAB = , EDC = por ser opuestos por el vrtice. Como el AF D o e externo en el BDF , se tiene AF D = + . Sumando los angulos internos del AEF se tiene + + + = 180. 3. (XV Competencia de Clubes Cabri Primera Ronda) En la gura 96, ABCD es un rectngulo tal que AB = 2BC. M es el punto medio de AB y los tringulos AM E a a y M BF son equilteros. Si P es la interseccin de las rectas DE y CF , encuentre los a o angulos del CDP . 72

Figura 96 Solucin: Note que AD = AE = F B = BC por lo que DAE y BCF son ambos o issceles. Luego DAE = CBF = 90+60 = 150 lo que implica que P DC = P CD = o 90 15 = 75 y luego CP D = 30. 4. Sea ABC un tringulo rectngulo con CAB = 90 (Ver gura 97). D es un punto sobre a a la prolongacin de BC tal que BD = BA. E es un punto en el mismo semiplano que A o respecto de BC, tal que CE BC y adems CE = CA. Mostrar que A, D y E estn a a alineados.

Figura 97 Solucin: Sea CBA = 2; el ABD es issceles y BAD + BDA = 2, por lo o o que BAD = BDA = . Como CAB = 90 entonces ACB + ABC = 90 y como CE es perpendicular a BC entonces ECA + ACB = 90; por lo tanto, ABC = ECA = 2. Con esto, como ECA es issceles, CEA = CAE = 90 . Luego, o EAC + CAB + BAD = 180 y as E, A y D estn alineados. a 5. Dado un cuadrado ABCD, se construyen los tringulos equilteros ABP (exteriormente) a a y ADQ (interiormente). Probar que C, P y Q estn alineados. Figura 98. a 73

Solucin: Observe que P AQ = BAD = 90 y P A = BA = DA = DQ, por lo o que P AQ es tringulo rectngulo issceles, y por tanto, P QA = 45. Por otra parte, a a o QDC = 90 ADQ = 30 y QD = AD = CD, es decir, el CDQ es issceles o con el angulo comprendido entre lados iguales de 30, por lo que DQC = 75. As , P QA + AQD + DQC = 180 y por lo tanto, C, P y Q estn alineados. a

Figura 98 6. En la gura 99, AB = BC = CD = DE = EF = F G = GA. Calcule la medida del DAE. Referenciasg20

Figura 99 Solucin: Sea DAE = . Como los tringulos ABC y AGF son issceles, ACB = o a o AF G = . Calculando los ngulos externos de ABC y AF G se tiene F BC = a CGF = 2. Como GF E y BCD son issceles, GEF = BDC = 2. Calculando o angulos externos de ADC y AEF se obtiene ECD = DF E = 3. Como CDE y F ED son issceles, CED = F DE = 3. Entonces, la suma de los angulos internos o del AED da + 3 + 3 = 180, de donde = 180 . 7 7. (XXVIII Olimpiada Brasilea de Matemtica) En la gura 100, AB = AC, AM = AN n a y CAM = 30, encuentre el valor del BM N . Solucin: Como ABC y AM N son issceles, sean ABC = ACB = y AM N = o o AN M . Por la frmula del angulo externo se tiene o

74

Figura 100

ACM + M AC = AM B = AM N + BM N + 30 = AN M + BM N = (N BM + BM N ) + BM N = + 2BM N Esto implica que BM N = 15. 8. (Etapa seminal Estatal de XXII Olimpiada Mexicana de Matemticas) En la gura a 101 se muestra un hexgono regular ABCDEF de lado 1. Los arcos del c a rculo que estn a dibujados tienen centro en cada vrtice del hexgono y radio igual a la distancia al vrtice e a e opuesto. P , Q, R, S, T y U son los puntos de corte de estos arcos. Cunto mide cada a lado del hexgono P QRST U ? a

Figura 101 Solucin: El hexgono P QRST U es regular y con el mismo centro que ABCDEF . Sea o a O el centro de ambos (Vase Figura 64). El lado buscado es igual a OP . Tenemos que e CF = F P = P C = 2 por ser radios de los arcos dibujados; entonces CF P es equiltero a de lado 2 y OP es una altura de este tringulo que, por Pitgoras, es igual a 3. a a

75

Congruencia de Tringulos a
1. En la gura 102, ABC es un tringulo equiltero y CDEF es un cuadrado. Se construye a a un punto G tal que CF = CG y adems CF G = 15. Probar que AGC = BDC. a

Figura 102 Solucin: BCD = 180 ACB DCF = 30. Como GCF es issceles, CGF = o o CF G = 15 y ACG = CGF + CF G = 30. Por criterio LAL, BCD ACG, por lo tanto BDC = AGC. 2. (Cuaderno de Olimpiadas Mexicanas - Geometr En la gura 103, ABCD un cuadrado a) y EF GH. Demuestre que que EF = GH.

Figura 103 Solucin: Se construyen EK y GM con K sobre CD y M sobre AD tales que EK o y GM CD. Luego se demuestra que EF K GHM , con EF = GH. AD

3. (Examen nal de XVI Olimpiada mexicana de Matemtica) Los angulos de un tringulo a a ABC estn en progresin aritmtica (B A = C B = ), D, E, y F son los a o e puntos medios de los lados BC, CA y AB, respectivamente. Llamamos H al pie de la altura trazada desde C (que cae entre B y F ) y G a la interseccin entre DH y EF . o Cunto vale F GH? a Solucin: Note que A + B + C = 3A + 3 = 180, lo cual implica que A + = o 76

60 = B. Entonces BCH es un tringulo 30, 60, 90, y dado que D es punto medio de a BC, el BDH es equiltero. Luego, como BC EG, F GH = BDH = 60. Ver gura a 104.

Figura 104 4. Sea ABCD un cuadrado. Se construyen tringulos equilteros ADP y ABQ como se a a muestra en la gura 105. Sea M la interseccin de CQ con AD y N la interseccin de o o CP con AB. Demuestre que CM N es un tringulo equiltero. a a

Figura 105 Solucin: Note que P D = AD, porque AP D es equiltero, y AD = CD porque o a ABCD es cuadrado, por lo que P D = CD, es decir, el CDP es issceles, con CDP = o CDA P DA = 30, entonces DP C = DCP = 75, y BCN = BCD DCP = 15. Anlogamente, BCQ es issceles con angulos 30, 75, 75, por lo que M CN = a o BCQ BCN = 60. Finalmente, como la gura es simtrica con respecto a AC, e CM = CN , entonces, el tringulo CM N es equiltero porque tiene dos lados iguales y a a un ngulo interno igual a 60. a 5. ABC es un tringulo issceles con ABC = ACB = 80. D es un punto en AC tal a o que ABD = 10. Demuestre que AD = BC. Figura 106. 77

Solucin: Se traza un punto D sobre AC tal que AD = BC. Se construye exteriormente o el tringulo equiltero AEB. Luego, AE = AB, D A = CB y EAD = ABC lo a a cual implica que EAD ABC, de donde se deduce que el D EB es issceles o y BED = BEA D EA = 40. Se sigue que EBD = 70 y como D BA = EBD ABE = 10, resulta que D = D y por lo tanto BC = AD = AD.

Figura 106

78

Cuadrilteros. a
1. Sea ABCD un paralelogramo. Se construyen tringulos equilteros exteriores CDP y a a ADQ, como se muestra en la gura 107. Demuestre que el BP Q es equiltero. a

Figura 107 Solucin: Observe que al hacer una rotacin de centro P y ngulo 60, el tringulo o o a a P BC se transforma en el tringulo P QD (observe los segmentos P C y CB tras esta a transformacin), mientras que al hacer una rotacin de centro Q y angulo 60, el tringulo o o a P QD se transforma en tringulo BQA. Como la rotacin mantiene las distancias, P B = a o P Q = BQ, por lo que el tr angulo BP Q es equiltero.33 a 2. (II Olimpiada Matemtica del Cono Sur) En la gura 108 ABCD y AECF son paralea logramos. Demuestre que BEDF es paralelogramo.

Figura 108
Una demostracin ms rigurosa se basa en el clculo de los ngulos P CB = P DQ = BAQ = 120 + o a a a ABC y en la utilizacin del criterio LAL para justicar P CB P DQ BAQ. o
33

79

Solucin 1: Sea M el punto medio de AC. Las diagonales AC y BD se bisecan en M , o mientras que las diagonales AC y EF tambin se bisecan en M , entonces BD y EF se e bisecan en M por lo que BEDF es un paralelogramo. Solucin 2: Como AD CB y AE CF entonces DAE = BCF . Entonces, por o propiedades de paralelogramos BAE = BAD EAD = BCD BCF = F CD; adems, AB = CD y AE = CF . Por criterio LAL, BAE DCF , y entonces BE = a DF . Anlogamente se demuestre que ABF CDE, lo cual implica BF = DE. a Como BEDF es un cuadriltero con lados opuestos iguales, es un paralelogramo. a 3. ABCD es un cuadriltero convexo y O es un punto en su interior. Sean P , Q, R, S, a los puntos medios de los lados AB, BC, CD, DA, respectivamente. Por P se traza una paralela a OR, por Q se traza una paralela a OS, por R se traza una paralela a OP , y por S se traza una paralela a OQ. Demuestre que estas cuatro rectas concurren. Solucin: Al tomar las rectas OP y OR y sus paralelas se forma el paralelogramo P ORM , o y al tomar las rectas OQ y OS y sus paralelas se forma el paralelogramo OQN S. Por el teorema de Varignon, sabemos que P QRS es un paralelogramo, y llamaremos T al punto de corte de sus diagonales. Observe que el punto de corte de las diagonales de P ORM es el punto medio de P R, i.e., T ; anlogamente, el punto de corte de las digonales de OQN S a es el punto medio de SQ, i.e., T nuevamente. As M es la reexin de O con respecto a , o T , y de igual forma queda denido N , por lo que M = N y las cuatro rectas concurren. 4. (Hctor Alberti) Sea ABCD un cuadrado. Se construyen los tringulos equilteros BDA , e a a ACB , BDC y ACD (Vase gura 109). Demuestre que el A B C D es tambin un e e cuadrado.

Figura 109 Solucin: Como A B = A D, AB = AD, CB = CD, C B = C D, los puntos A , A, o C, C pertenecen a la mediatriz de BD, y por tanto, estn alineados. Anlogamente, B , a a B, D, D estn alineados; por lo tanto A C B D . Por otra parte, si O es el centro a de ABCD, como los tringulos equilteros construidos son todos iguales (tienen lados a a iguales a la diagonal de ABCD) de altura h, OA = OB = OC = OD = h. Entonces, A B C D es un cuadriltero con diagones que se bisecan en O (es paralelogramo), son a iguales A C = B D = 2h (es rectngulo) y son perpendiculares (es rombo), lo cual a implica que es cuadrado. 80

5. Un trapecio issceles tiene diagonales perpendiculares y su area es 2010, determine su o altura. Solucin: Considere la gura 110. Sea ABCD el trapecio del problema (AB = CD), o como es trapecio issceles, es simtrico con respecto a la mediatriz de las bases, en partio e cular, AC = BD. Sean P y Q los pies de las perpendiculares a AD trazadas desde B y C, respectivamente. Por LAL, ABD DCA lo cual implica CAD = BDA = 45 (debido a que AC BD) luego en el tringulo rectngulo ACQ, ACQ = 45 por lo a a que AQ = CQ y es fcil ver que BC = P Q. Luego a 2010 = (ABCD) = (CQ)(BC + AD) 2 (CQ)(P Q + AP + P Q + QD) = 2 = (CQ)(AP + P Q) = CQ2

Luego CQ =

2010.

Figura 110 6. (IX Competencia de Clubes Cabri, Segunda Ronda) Sea ABCDEF un hexgono regular a cuyo centro es O. Se construyen los cuadrados F SOP y ORCQ. Demuestre que AP QB y SEDR son rectngulos. Figura 111. a Solucin: Por construccin P F = P O = SF = SO, y por propiedades de hexgono o o a 34 regular AF = AO = EF = EO, entonces P , S A, E, pertenecen a la mediatriz de F O y por tanto, estn alineados sobre una recta perpendicular a F O. Anlogamente, Q, R, a a B, D estn alineados sobre una recta perpendicular a CO; adems, es fcil demostrar a a a que AP = BQ = DR = ES. Observe adems que AB CF a DE, lo cual implica (AB CF ) (AP BQ), es decir, AP QB es rectngulo, y anlogamente para SEDR. a a

34

Los tringulos OAB, OBC, OCD, ODE, OEF , OF A son equilteros. a a

81

Figura 111 7. Sobre los lados del ABC se trazan exteriormente los cuadrados ABP Q, CARS y BCT U . Luego se trazan los paralelogramos AQA R, CSC T y BU B P , como en la gura 112. a) Sean A , B , C los centros de los cuadrados BCT U , CARS, ABP Q, respectivamente. Demuestre que estos centros estn sobre los lados del A B C . a b) Demuestre que AA , BB , CC concurren. Solucin: o a) Observe que al hacer una rotacin de centro A y angulo igual a 90, el A U B se o transforma en el A BA, y a la vez este ultimo se transforma en el A CC (esto es porque A U A B A C y U B BA CC ); esto signica que A B A A y A A A C , por lo que B , A , C estn alineados, es decir, A pertenece a B C . a Anlogamente se prueban los otros casos. a b) De lo anterior, observe que AA es mediatriz de B C , por lo que AA , BB , CC concurren en el circuncentro del A B C . 8. Se dibujan cuadrados exteriores a los lados de un paralelogramo (Vea gura 113), demuestre que: a) El cuadriltero determinado por los centros de esos cuadrados es un cuadrado. a b) Las diagonales de ese cuadrado son concurrentes con las del paralelogramo. Solucin: o

82

Figura 112 a) Observe que al hacer una rotacin de centro O2 y angulo igual a 90, el O2 BO1 se o transforma en el O2 CO3 (observe que los segmentos O2 B y BO1 se transforman en O2 C y CO3 , respectivamente), por lo que O2 O1 = O2 O3 y O2 O1 O2 O3 . Repitiendo este razonamiento, O1 O2 = O2 O3 = O3 O4 = O4 O1 y estos segmentos son perpendiculares si son consecutivos, por lo que O1 O2 O3 O4 es un cuadrado. b) Basta demostrar que AC y O1 O3 se bisecan,35 y esto es equivalente a demostrar que AO1 CO3 es un paralelogramo. Esto es cierto porque AO1 = CO3 y AO1 CO3 (ambos segmentos son perpendiculares a O1 B)

Figura 113
35

Porque as los puntos de corte de las diagonales de ABCD y O1 O2 O3 O4 coincidir an.

83

9. Dado un ABC, se construyen exteriormente los tringulos rectngulo issceles ACP a a o y BCQ, con AC y BC como hipotenusas. Si M es el punto medio de AB, demuestre que el M P Q tambin es un tringulo rectngulo issceles. e a a o Solucin: Construya los cuadrados exteriores ACDE y BCF G, como muestra la gura o 114. Observe que P y Q son los puntos medios de AD y BF , respectivamente. Al rotar el BCD con centro C y angulo de 90, se genera el F CA, entonces dichos tringulos a son congruentes y en por tanto BD = AF y BD AF . Por otra parte, observe que M P es base media del BAD, por lo que 2M P = BD y M P BD; anlogamente, M Q es a base media del ABF , por lo que 2M Q = AF y M Q AF . Por lo tanto M P = M Q y M P M Q.

Figura 114

84

Angulos en Circunferencia.
1. Dada la gura 115, demuestre que AB AB.

Figura 115 Solucin: Observe que los cuadrilteros ABQP y A B QP son c o a clicos, por lo que P AB = P QB = 180P A B , por lo tanto AB A B . 2. Dos circunferencias de centros O1 y O2 son tangentes (interna o externamente) en un punto P ; por este punto se traza una recta que corta nuevamente a la circunferencias en A y B, respectivamente. Demuestre que AO1 BO2 . Solucin: En la gura 116 se ha considerado que las circunferencias son tangentes exteo riormente, sin embargo, el otro caso es anlogo. Se sabe que O1 , P , O2 estn alineados, y a a que AP O1 y BP O2 son tringulos issceles (dos de sus lados son radios de una circuna o ferencia), entonces O1 AP = AP O1 = BP O2 = O2 BP , por lo que AO1 BO2 .

Figura 116 3. Dadas dos circunferencias una fuera de la otra como en la gura 117, demuestre que las tangentes comunes externas forman segmentos iguales; anlogamente, las tangentes coa munes internas forman segmentos iguales. Solucin:36 Sea P la interseccin de las tangentes comunes externas AA y BB . Entonces o o
Suponemos que las circunferencias tienen radios distintos; cuando los radios son iguales, el problema se justica por la simetr de la gura. a
36

85

AA = P A P A = P B P B = BB . Anlogamente se resuelve el caso de las tangentes a comunes internas.

Figura 117 4. Teorema de Pithot. Demuestre que en todo cuadriltero inscribible, la suma de lados a 37 opuestos es igual. Solucin: Considere la gura 118, ABCD es el cuadriltero inscribible, con P , Q, R, S, o a los puntos de tangencia sobre AB, BC, CD, DA, respectivamente. Entonces AB + CD = AP + P B + CR + RD = AS + BQ + CQ + DS = BC + DA

Figura 118: Teorema de Pithot. 5. Teorema de Steiner. En todo cuadriltero exinscrito a una circunferencia, la diferencia a de las longitudes de lados opuestos es igual. Solucin: El cuadriltero puede quedar en posiciones como las de la gura ??; en ambos o a casos, la demostracin es muy similar, y anloga a la de Pithot. Para la gura de la o a
37

El rec proco de este teorema y del siguiente son tambin es ciertos. e

86

izquierda se tiene que AB CD = (AP BP ) (CR RD) = (AS BQ) (CQ DS) = AD BC

Figura 119: Teorema de Steiner. 6. Teorema de Miquel: Dado un ABC, sean X, Y , Z puntos sobre AB, BC, CA, respectivamente . Demuestre que los circunc rculos de AXZ, BY X, CZY tienen un punto en comn M . u Solucin: Sea M el otro punto de corte de los circunc o rculos de los cuadrilteros AXM Z y BY M X son c a clicos, se tiene AXZ y BY X. Como

Y M Z = 360 XM Z Y M X = 360 (180 A) (180 B) = 180 C Entonces, CY M Z es cuadriltero c a clico, por lo que M est sobre el circunc a rculo del CZY .

Figura 120: Teorema de Miquel.

87

7. Sea ABC un tringulo, y sean L y N las intersecciones de la bisectriz del angulo A con a el lado BC y el circunc rculo de ABC respectivamente (Ver gura 121). Construimos la interseccin M del circunc o rculo de ABL con el segmento AC. Prueba que los tringulos a BM N y BM C tienen la misma area. Solucin: Observe que ABN C y ABLM son cuadrilteros c o a clicos, por lo que N CB = N AB = LAM = LBM , por lo que CN BM . Entonces, las distancias de N y C a la recta BM son iguales, y por tanto, el area del BM N es igual al area del BM C.

Figura 121 8. Sea AB el dimetro de una semicircunferencia. Se colocan los puntos M y K sobre la a semicircunferencia y sobre AB, respectivamente.38 Sea P el centro de la circunferencia que pasa por A, K y M ; sea Q el centro de la circunferencia que pasa por B, K y M . Demuestre que M P KQ es conc clico. Solucin: Como AB es dimetro, AM B = 90, entonces M AB + M BA = 90. As o a M P K + M QK = 2M AK + 2M BK 2 = 2 (M AB + M BA) = 180

Por lo tanto, M P KQ es conc clico.

38

M y K son distintos de A y B.

88

9. Las circunferencias 1 y 2 se cortan en los puntos A y B. Por el punto A se traza una recta que corta nuevamente a las circunferencias 1 y 2 en los puntos C y D, respectivamente. Por los puntos C y D se trazan tangentes a las circunferencias, las cuales se cortan en el punto M . Demuestra que M CBD es c clico. Figura 122. Solucin: Es suciente probar que M CBD es un cuadriltero con un par de ngulos o a a opuestos suplementarios. Por angulos seminscritos y suma de angulos internos de un tringulo, se tiene a CM D + CBD = CM D + CBA + DBA = CM D + M CA + M DA = 180

Figura 122 10. El ABC cumple que A = 90 y AB = AC. Se toma un punto E del segmento AB, se construye interiormente un tringulo equiltero AEF . EF corta BC en I, y se construye a a exteriormente un tringulo equiltero BIJ. Encuentre EJB. a a

Solucin: Como el BJI = 60 = AEI, o el cuadriltero BEIJ es c a clico, por lo que el EJB = EIB = AEI EBI = 15.

89

11. En la gura 123, se sabe que AO1 B AO2 B = 70 y adems la tangente EB forma a el tringulo issceles ABE, con AB = AE. Encuentre EBC. a o

Figura 123 Solucin: Sea AO2 B = 2, entonces ACB = y por hipteis AO1 B = 2 + 70. Por o o angulo seminscrito, ABE = + 35, y como el ABE es issceles, AEB = + 35. o Finalmente, por la frmula del ngulo externo aplicada al BCE, EBC = AEB o a ECB = 35 12. Dos circunferencias 1 y 2 se cortan en A y B. Una recta por A corta a 1 y 2 en C y D, respectivamente, y la paralela a CD por B corta 1 y 2 en E y F , respectivamente. (Ver gura 124). Demuestre que CDB EAF . Solucin: Sean G = AE BC y H = AF BD. Como AC BE y ACEB es c o cli39 co, CAG = GEB = ACG = GBE = ; anlogamente, DAH = HF B = a ADH = F BH = . Observe que GAC y GBE son tringulos issceles y por tana o to AE = AG+GE = CG+GB = CB; de forma similar se obtiene AF = DB. Finalmente, EAF = 180 = CBD, por lo que, por el criterio LAL, CBD EAF . 13. La Recta de Simson-Wallace. Sean X, Y y Z los pies de las alturas trazadas desde un punto P en el circunc rculo del ABC hacia AB, BC y CA, respectivamente. Demuestre que X, Y y Z estn alineados.40 a Solucin: Como BP Y X es c o clico, Y XP = Y BP = . Como ABP C es c clico, CBP = CAP = . Como AXP Z es c clico, ZAP = ZXP = . Por lo tanto, dado que Y XP = ZXP , los puntos X, Y y Z estn alineados. a
ACEB es un trapecio issceles. o El rec proco tambin es cierto, si X, Y y Z estn alineados, entonces P debe estar sobre el circunc e a rculo del ABC; en cualquier otro caso, el XY Z se llama el tringulo pedal con respecto al punto P . a
40 39

90

Figura 124

Figura 125: Recta de Simson-Wallace 14. Sea P un punto exterior al cuadrado ABCD tal que AP C = 90 , Q es la interseccin o de AB y P C, y R el pie de la perpendicular por Q a CA. Demuestre que P , R y D estn a alineados. Solucin: Como AP C + ADC = 180, el cuadriltero P ADC es c o a clico, entonces AP D = ACD = 45. Anlogamente, como AP Q + ARQ = 180, el cuadriltero a a P ARQ es c clico, entonces AP R = AQR = 90QAR = 45. Por lo tanto, como AP D = AP R, los puntos P , R, D estn alineados. Figura 126. a 15. (OIM 2002, P-4) En un tringulo escaleno ABC se traza la bisectriz interior BD, con D a sobre AC. Sean E y F puntos sobre la recta BD tales que (AE CF ) BD, y sea M el punto sobre el lado BC tal que DM BC. Demuestre que EM D = DM F . Figura 127. 91

Figura 126 Solucin: Como DM M C y DF F C, DF CM es c o clico, por lo tanto DM F = DCF = , y como AE F C, entonces EAD = DCF = . Sea G la interseccin o de AE con BC. Como AG BE, BE es altura y bisectriz del ABG, por lo que este tringulo es issceles y adems BE es mediatriz de AG; entonces EGD = EAD = . a o a Y nalmente, podemos ver que DEM G es c clico, pues DEG = DM G = 90 , as que EM D = EGD = . De aqu el resultado es inmediato. ,

Figura 127 16. (OMCC 2003, P-2) Sea S una circunferencia y AB un dimetro de ella. Sea t la recta a tangente a S en B y considere dos puntos C y D en t tales que B este entre C y D. Sean E y F las intersecciones de S con AC y AD y sean G y H las intersecciones de S con CF y DE. Demuestre que AH = AG. Solucin: Como AEBF es c o clico (Ver gura 128), AEF = ABF . Luego, como AB CD y BF AD, se cumple tambin ABF = F DB, por lo que AEF = F DC, es e decir, el cuadriltero CDF E es c a clico. Utilizando este resultado y el hecho que EGHF tambin es c e clico, se tiene EDC = EF G = EHG, por lo que CD GH. Esto 92

implica que AB GH, y como AB pasa por el centro de la circunferencia, debe ser mediatriz de GH, por lo tanto AG = AH.

Figura 128 17. (The 59th Romanian Mathematical Olympiad District Round) Considere un cuadrado ABCD y un punto E sobre el lado AB. La diagonal AC corta al segmento DE en el punto P . La perpendicular por P a DE corta al lado BC en F . Probar que EF = AE + CF . Solucin: Se construye E sobre BC de tal manera que CE = AE (como se mueso tra en la gura 129) y que C quede entre F y E , as por LAL se tiene que los tringulos a rectngulos DAE DCE por lo tanto ADE = CDE luego EDE = 90. Por a otra parte, el cuadriltero DCF P es c a clico, por lo que P DF = P CF = 45 entonces F DE = EDE EDF = 45. Ahora por LAL los tringulos DEF DE F , a por lo que EF = E F = E C + CF = AE + CF .

Figura 129

93

18. Teorema de Arqu medes: En la gura 130, la regin delimitada por tres semicircunfeo rencias mutuamente tangentes, es conocida como cuchilla de zapatero o rbelos. Demostrar a que las circunferencias sombreadas son congruentes.

Figura 130: Teorema de Arqu medes. Solucin: Sean AB, AC, BC los dimetros de las semicircunferencias que forman el arbeo a los, de radios r, r1 , r2 y centros O, O1 , O2 , respectivamente. De momento nos concentramos en el lado izquiero de la gura Referenciasgura58; sea C1 el centro de la circunferencia de la izquierda y R1 su radio; D, E y F son los puntos de tangencia de esta circunferencia con dos semicircunferencias del rbelos y con la recta perpendicular a los dimetros por C; a a nalmente, G es la proyeccin de C1 sobre AB. En primer lugar, OO1 = OAO1 A = rr1 . o Por otra parte, observe que O, C1 , D estn alineados y O1 , C1 , E tambin estn alineados, a e a entonces OC1 = OD C1 D = r R1 y O1 C1 = O1 E + EC1 = r1 + R1 . Adems, como a CF C1 G es un rectngulo, GC = F C1 = R1 , entonces O1 G = O1 C GC = r1 R1 y a OG = O1 G O1 O = (r1 R1 ) (r r1 ) = 2r1 (r + R1 ). Ahora, aplicando el teorema de Pitgoras a GO1 C1 y GOC1 se tiene a
2 2 O1 C1 O1 G2 C1 G2 = OC1 OG2 C1 G2 (r1 + R1 )2 (r1 R1 )2 = (r R1 )2 (2r1 (r + R1 ))2 2 4r1 R1 = 4rR1 4r1 + 4r1 r + 4r1 R1 r1 (r r1 ) R1 = r

Figura 131 Anlogamente, si r2 y R2 son los radios del semic a rculo y del c rculo de la derecha, respectivamente, entonces r2 (r r2 ) R2 = r 94

Pero 2r = AB = AC + BC = 2r1 + 2r2 , entonces r2 = r r1 , y sustituyendo en la ecuacin anterior se tiene o R2 = (r r1 ) (r (r r1 )) (r r1 )r1 = = R1 r r

95

Teorema de Thales y Semejanza.


1. (IV OMCC, P-4) Sea ABC un tringulo, D el punto medio de BC, E un punto sobre a el segmento AC tal que BE = 2AD y F el punto de interseccin de AD con BE. Si o CAD = 60, encuentre la medida de los angulos del F EA. Figura 132. Solucin: Se traza por D una paralela a BE y sea G el punto por el que esta paralela o corta al lado AC. Como DG es base media del BCE se tiene que DG = BE = AD; 2 entonces ADG es issceles y tiene un agulo de 60, por lo que debe ser equiltero. o a Finalmente AEF tambin es equiltero, por tanto sus ngulos son iguales a 60. e a a

Figura 132 2. Sea ABCD es un trapecio con AD BC. M y N son los puntos medios de CD y BC, respectivamente, y P el punto comn de las rectas AM y DN . Si P M = 1 , demuestre que u AP 4 ABCD es paralelogramo. Solucin: Sea Q el punto medio de DN , entonces QM o BC DA. Como M Q es P AD, base media del CDN , M Q = CN = CB . Por otra parte, como P M Q 2 4 MQ PM 1 AD = AP = 4 , entonces M Q = 4 . Finalmente, como BC DA y BC = DA, ABCD AD es paralelogramo. 3. En la gura 133, BC = CD = DE = EA = x y AEB = 90. Demuestre que ABC + ACD + ADE = 90.

Figura 133 Solucin 1: Por Pitgoras, AD = 2x. Observe que DA2 = 2x2 = DB DC, por lo que o a ABD CAD; entonces ABD = CAD y por tanto ABC + ACD + ADE = 96

CAD + ACD + ADE = 2ADE = 90. Solucin 2: Considere la siguiente cuadr o cula (Figura 134). Observe que al hacer una rotacin de centro A y ngulo igual a 90, el segmento AC se transforma en AF , por lo o a que el ACF es tringulo rectngulo issceles, y ADE = ACF . Se cumple ABC = a a o CF D, porque se forman con la diagonal de tres cuadrados; anlogamente, ACD = a AF D, porque se forman con la diagonal de dos cuadrados. Sumando los angulos internos del ACF se obtiene el resultado buscado.

Figura 134 4. (Asitico Pac a ca) Sea ABC un tringulo y D el pie de la altura con respecto a A. Sean E a y F puntos en una recta que pasa por D (distintos de D) tales que AE CE y AF BF . Sean M y N los puntos medios de BC y EF , respectivamente. Demuestre que AN N M . Solucin: En la gura 135, AE CE y AD DC entonces, ADEC es c o clico, as que DEA = DCA. Del mismo modo, como AF BF y AD DB, AF BD es c clico y entonces AF D = ABD. Esto implica que ABC AF E y a partir de esta semejanza, ABM AF N . Luego, AM B = AN F , por lo que el cuadriltero a AN DM es c clico, y por lo tanto AN M = ADM = 90.

97

Figura 135

Puntos y rectas notables.


1. De acuerdo con los datos de la grca 136, calcular el valor de AB. a

Figura 136
CD 5 Solucin 1: Por el teorema de la bisectr AB = DB , de donde AC = 4 x, luego, aplicando o z AC 2 el teorema de pitgoras al ABC, se tiene que x2 + 182 = 5 x , que despus de resolver a e 4 se tiene que x = 24.

Solucin 2: Dado que D es un punto de la bisectr del BAC, entonces D equidista o z de los lados de dicho angulo, sea pues H AC talque DHAC y DH = DB = 8 entonces, aplicando el teorema de pitgoras en el CDH se deduce que HC = 6, por lo a que AC = x + 6, y por el teorema de pitgoras en el ABC, x2 + 182 = (x + 6)2 , por lo a tanto, x = 24. 2. Sea ABCD un paralelogramo. Q es el punto medio de AD, F el pie de la perpendicular por B sobre QC. Probar que AF = AB. Solucin: Sea E el punto medio de BC y G la interseccin de AE con BF . Como o o AE CQ, se tiene que AG BF . Pero tambin, como AE CQ, entonces EG CF e por lo que en el BCF , EG es base media. Entonces BG = GF de donde se sigue que ABF es issceles porque BG es altura y mediana. o 98

3. Sea ABCD un cuadriltero tal que AB = CD. Las mediatrices de AC y BD se cortan a en P . Probar que P AC = P CA = P BD = P DB. Figura 137. Solucin: Como P est sobre las mediatrices de AC y BD, P A = P C y P B = P D, o a y por hiptesis, AB = CD, entonces por criterio LLL, ABP CDP . De aqu o , AP B = CP D, entonces BP D = AP D + AP B = AP D + CDP = AP C; por lo tanto, BP D CP A, dada la igualdad anterior y el hecho que son tringulos a issceles. De esta semejanza se obtiene P AC = P CA = P BD = P DB. o

Figura 137 4. ABC es un tringulo y P un punto en su interior. Sean A , B y C las reexiones de a P sobre BC, CA y AB, respectivamente. D, E y F son los pies de las perpendiculares respectivos desde A, B y C hacia B C , C A y A B . Probar que AD, BE y CF son concurrentes. Figura 138. Solucin: Por propiedades de reexin axial AC = AP = AB , por lo que el AB C es o o issceles, y entonces AD es mediatriz de B C . Anlogamente, BE es mediatriz de C A , o a mientras que CF es mediatriz de A B . Por lo tanto, las rectas AD, BE, CF concurren en el circuncentro del A B C . 5. (Arnoldo Aguilar) En la gura 139, ABGH, BCF G y CDEF son cuadrados. Si I es el centro de ABGH y J = DH BG, demuestre que I, J y F estn alineados. a Solucin: Como G es punto medio de HF , BG es una mediana del BF H. Adems, o a BDF H es un paralelogramo, luego sus diagonales BF y DH se cortan en su punto medio, digamos K. Se sigue que HK es tambin una mediana del BF H, y en consecuencia el e punto de corte de J = KH BG es el centroide del BF H. Pero I es el punto medio de BH, as que F I es la tercera mediana del BF H, por lo tanto J est sobre el segmento a F I.

99

Figura 138

Figura 139 6. (Arnoldo Aguilar) Sea ABC un tringulo equiltero. M y N son los puntos medios de AB a a y BC, respectivamente. Exteriormente al ABC se construye un tringulo rectngulo a a issceles AP C, con AP C = 90 . Si I es la interseccin de AN y M P , demuestre que o o CI es la bisectriz de ACM .

Solucin: Observe que AN es bisectriz del BAC. o Como AP C = BM C = 90, el cuadriltero a AP CM es c clico, por lo que P M C = P AC = P CA = P M A = 45, entonces M P es bisectriz del AM C. De aqu se concluye que I = M P AN es el incentro del ACM , por lo que CI es bisectriz del ACM .

7. En la gura 140, el circunferencia?

ABC es tal que A = 90 y B = 60. Cul es el radio de la a

Solucin: Por relaciones de tringulos notables, BC = 2 y CA = 3. Sean P y Q o a las proyecciones de O sobre AB y AC respectivamente; por construccin, AP OQ es un o rectngulo, pero como OP = r = OQ, es tambin cuadrado, por lo que AP = r. Observe a e 100

Figura 140 que la circunferencia es el exc rculo del ABC, por lo que AP = s y entonces

r = s 1+2+ 3 = 2 3+ 3 = 2 8. Dado el paralelogramo ABCD, sea M el punto medio de AB, y N la interseccin de CD o con la bisectriz interna del ABC. Demuestre que M C BN si y slo si AN es bisectriz o del DAB. Solucin: o () Si suponemos que M C BN entonces BN es mediatriz de M C, y como BM CN entonces CBN = M BN = CN B = M N B, esto implica que BC M N , y por tanto N es punto medio de CD; as AM N D es un rombo y AN es bisectriz del DAM . , () Si suponemos que AN es bisectriz del DAB, es propiedad conocida que AN BN , por lo que M es el circuncentro del ABN y por la relacin entre angulo central y ngulo o a inscrito se tiene AM N = 2ABN = ABC, por lo tanto M N BC y BCN M es un rombo, de donde se obtiene M C BN . 9. En el ABC, se sabe que los vrtices B, C, el circuncentro O y el ortocentro H del e ABC estn todos sobre una misma circunferencia. Figura 141. a a) Calcule el valor de A. b) Demuestre que el incentro tambin pertenece al circunc e rculo de BCOH. Solucin: o a) Sea BAC = . Como O es el circuncentro del ABC, tenemos que BOC = 2. Por otra parte, sabemos que al ser H ortocentro, se cumple que BHC = 180 . Ahora 101

bien, la condicin de que B, C, H y O son conc o clicos implica que BOC = BHC, de donde 2 = 180 , y por tanto = 60 .

Figura 141 b) Este problema se basa en el siguiente resultado: si I es el incentro del ABC entonces BIC = 90 + A . Como en este caso A = 60, entonces BIC = 120 = BOC = 2 BHC, por lo que B, C, O, H, I, se ubican sobre una misma circunferencia. 10. Sea ABC un tringulo tal que las medianas respectivas a B y C son perpendiculares. a Demuestre que se cumple la relacin (Ver gura 142). o 5BC 2 = CA2 + AB 2 .

Figura 142 Solucin: Sean BB y CC las medianas que son perpendiculares, y sea G el centroide. o Observe que el cuadriltero BCB C tiene diagonales perpendiculares; por el teorema de a Pitgoras se cumple a BC 2 + B C 2 = C B 2 + B C 2 2 2 BC AB AC 2 = + BC + 2 2 2 2 2 2 5BC = AB + AC 102
2

11. Sea ABC un tringulo de ortocentro H. Sean P y Q los pies de las perpendiculares desde a H a las bisectrices interior y exterior de A, respectivamente. Si M es el punto medio de BC, mostrar que P , Q y M estn alineados. Figura 143. a Solucin: Sean E y F los pies de las alturas trazadas desde a B y C, respectivamente. o Se sabe que AP AQ, por lo que AP HQ es un rectngulo. Como AP H = AQH = a AEH = AF H = 90, los puntos P , Q, E, F , pertenecen a una circunferencia de dimetro AH. Adems, en esta circunferencia, como AP y AQ son bisectrices (interior a a y exterior, respectivamente) del EAF , P y Q son los puntos medios del arcos EF , por lo que P Q es la mediatriz de EF . Por otra parte, como BEC = BF C = 90, el cuadriltero BCEF es c a clico, y el circuncentro es M , por lo que M E = M F ; entones M est en la mediatriz de EF , la cual es P Q. a

Figura 143 12. En un tringulo ABC, sea M el punto medio de BC. Si se cumple que AB = AC y a adems M AC + ABC = 90 , hallar BAC. Figura 144. a Solucin: Sin prdida de generalidad, suponga que AB > AC. Sea N la interseccin de o e o AB con la mediatriz de BC. Se forma el BCN que es issceles, entonces CN M = o 90 M CN = 90 M BN = CAN , lo cual implica que el cuadriltero ACM N es a c clico. Por lo tanto, BAC = BM N = 90. 13. Sea ABC un tringulo y U un punto de su circunc a rculo tal que AU es bisectriz. Las mediatrices en AB y AC cortan a AU en X y Y . Sea T la interseccin de BX con CY . o Demostrar que AU = T B + T C. Figura 145. Solucin:41 Como X y Y pertenecen a las mediatrices de AB y AC, respectivamente, o y a la bisectriz AU , entonces ABX y ACY cumplen ser issceles y semejantes entre o si, porque XBA = XAB = Y AC = Y CA = . Esto implica T XY = XBA +
41

El caso cuando AB = AC es trivial, porque X, Y y T colapsan en el circuncentro del

ABC.

103

Figura 144 XAB = 2 = Y AC+Y CA = T Y X, es decir, el T XY es issceles con T X = T Y . o Por otra parte, como ABU C es c clico, U BC = U AC = U AB = U CB = . De aqu se concluye que U BC es issceles, con U B = U C. Adems, XU B = ACB = o a Y CU y XBU = ABC = Y U C; por criterio ALA, U XB CY U , por lo que BX = Y U . Finalmente, T B + T C = (BX T X) + (CY + T Y ) = Y U + AY = AU .

Figura 145 14. (The 59th Romanian Mathematical Olympiad Final Round) Sea ABCD un rectngulo de a centro O con AB = BC. La perpendicular en O a BD corta a las l neas AB y BC en los puntos E y F , respectivamente. Sean M y N los puntos medios de los segmentos CD y DA, respectivamente. Probar que las l neas rectas F M EN . Solucin: Considere la gura 146, sin prdida de generalidad, se ha supuesto AB < BC.42 o e Sea L el punto medio de AB, y H es la interseccin de EF con AD. Se tiene que o LN BD, y como BD EF entonces LN EF ; adems, como ABCD es un rectngua a lo, DA AB, por lo tanto, H es el ortocentro del ELN , y as LH EN . Por otra , parte, las reexiones de L y H con respecto a O son respectivamente M y F , por lo que LH M F , lo cual implica que F M EN .
42

El otro caso es completamente anlogo. a

104

Figura 146 15. Sea ABC un tringulo rectngulo, con A = 90 . Sea D un punto en su interior tal que a a DAC = DCA = DBC = , y AC = BD. Determine el valor de . Figura 147. Solucin: Sean P y Q los pies de las perpendiculares trazadas desde D hacia CA y AB, o respectivamente, R es un punto sobre BC tal que DB DR, y E es la interseccin de CD o con AB. Como el ACD es issceles, P es punto medio de AC, entonces AC = 2P A = o 2DQ = BD, por lo que el BDQ es un tringulo notable y DBQ = 30. Por otra parte, a por criterio ALA, ACE DBR, por lo que CE = BR; como P D AE, D es punto medio de CE; as si M es el punto medio de BR (y circuncentro del BDR) se cumple , que DC = RM = DM , por lo que el CDM es issceles. Por la relacin entre el angulo o o inscrito y el angulo central DM R = 2DBR, por lo tanto DCR = 2. Sumando los angulos internos del ABC se tiene A + B + C = 90 + 30 + + 3 = 180, lo cual implica = 15.

Figura 147 16. Sea ABC un tringulo y M un punto tal que M AB = 10, M BA = 20, M AC = 40 a 105

y M CA = 30. Probar que el ABC es issceles. Figura 148. o Solucin: Sea D la reexin del punto A con respecto a la recta BM . Entonces el AM D o o es issceles con AM D = 2 (M AB + ABM ) = 60 y por lo tanto es equiltero. Tamo a bin DBA = 2M BA = 40 y como BAC = 50, implica que DB AC. Sea E la e interseccin de BD con CM , se cumple que CED = 90 ACE = 60 = M AD, o por lo que el cuadriltero AM ED es c a clico. De aqu DEA = DM A = 60. Como , DEC = DEA y ED AC, se tiene que ED es bisectriz y altura en el AEC, por lo tanto ED es mediatriz de AC, lo cual implica que BA = BC.

Figura 148 17. Teorema de Poncelet: Demuestre si entonces 2(r + R) = b + c. ABC es un tringulo rectngulo con A = 90, a a

Figura 149: Teorema de Poncelet Solucin: Sean O e I el circuncentro y el incentro del ABC. Como A = 90, O es el o punto medio de BC, por lo que a = 2R. Por otra parte, si P y Q son las proyecciones de I sobre AB y AC, claramente AP IQ es rectngulo, pero como I es incentro IP = r = IQ, a por lo que AP IQ es cuadrado. Se sabe que para un tringulo cualquiera AP = s a, por a lo tanto r = sa b+ca r = 2 2r + a = b + c 2(r + R) = b + c 106

18. En la gura 150, ABCD y P QRS son cuadrados, ABP BCQ CDR DAS y los los radios de las cinco circunferencias son iguales a r. Si a es el lado del cuadrado ABCD, determine r en funcin de a. o

Figura 150 Solucin: Se tiene AB = a y se denen b = AP y c = BP ; observe que por las congrueno cias BQ = b, por lo que P Q = cb = 2r. Por otra parte (anlogamente a la demostracin a o del teorema de Poncelet), al calcular el inradio del ABP se tiene que 2r = b + c a, entonces c b = b + c a, lo cual implica que a = 2b. Por lo tanto, el ABP es un tringulo notable de 30, 60, 90, y as a r = cb 2 3 a 1a 2 = 2 2 31 = a 4

107

19. Recta de Euler. El centroide G, el ortocentro H y el circuncentro O de un tringulo a estn alineados, y adems GH = 2GO. a a

Figura 151: Recta de Euler Solucin: Considere la siguiente gura. Sean AHa y BHb alturas, OA y OB mediatrices. o Observe que HA BC y OA BC, por lo que HA OA ; anlogamente HB OB ; a tambin, por el teorema de la base media AB A B y AB = 2A B . Esto implica que e ABH A B O y la razn de semejanza es 2; en particular AH = 2A O. Si denimos o G como la interseccin de la mediana AA con HO, claramente AHG o A OG y la razn de semejanza es la misma que la anterior, por lo que GA = 2GA , i.e., G es el o centroide del ABC. Esto implica que el ortocentro, el centroide y el circuncentro de un tringulo estn alineados, y por la semejanza GH = 2GO. a a

108

Вам также может понравиться